Download PGI PG 2020 May Surgery Solved Question Paper

Download PGIMER (Post Graduate Institute of Medical Education & Research, Chandigarh) 2020 May Surgery Solved Question Paper

1.Inapatientwithilealresection,ileoc-coecal
valesarespared.Whichofthefollowing
candevelop?

a)Bacterialovergrowth
b)Malabsorption
c)Steatorrhea
d)Cholelithiasis
e)Renaloxalatestones
CorrectAnswer-B:C:D:E
Answer-B,C,D,E
Patientswithshort-bowelsyndromeinvariablypresentwithahistory
ofseveralintestinalresections.
Malabsorption,diarrhea(withorwithoutsteatorrhea)isanalmost
constantclinicalfindings.
Terminalileumresection-waterydiarrhea/steatorrhea,
malabsorbtion,megaloblasticanemia,oxalatekidneystones,
cholesterolgallstones.

2.Alcoholisariskfactorforwhichofthe
followingcarcinoma(s)?
a)Esophagus
b)Liver
c)Pancreas
d)Cervix
e)Larynx
CorrectAnswer-A:B:C:D:E
Answer-A,B,C,D,E
Alcoholmoststronglyincreasedtheriskforcancersoftheoral
cayity,pharynx,esophagusandlarynx.
Significantincreasesinriskalsoexistedforcancersofthestomach,
colon,rectum,liver,femalebreast,andovaries.
Alcoholicwomenareathighriskforinsituandinvasivecervical
cancerandforcancerofthevagina.

3.Trueregardingesophagealsquamouscell
carcinomais/are?
a)Barrett'sesophagusisariskfactor
b)Commoninmiddlethirdofesophagus
c)Stomach,jejunumorcoloncanbeusedforreplacementafter
surgicalremoval
d)Chemoradiationhaslittleroleininoperablepatients
e)StagingisdonebyCECT
CorrectAnswer-B:C:E
Answer-B,C,E
"ThestomachremainsthemostPreferredesophagealsubstitute
followingesophagectomyforcancer.
Squamouscellcarcinoma-
Itisthemostcommontypeofesophagealcarcinomaworldwideand
inIndia.Itusuallyoccursinmiddle1/3rd(most
common)andupper1/3rdofesophagus.Somemayalsoarisein
lower1/3rd.
Importantriskfactors-
Plummer-Vinson-PatersonKellysyndrome
Tylosisplamarisetplantaris
Humanpapillomavirus(HPV)infection
Flexibleendoscopywithbiopsyistheprimarymethodfordiagnosis
ofesophagealcancer.
CECTchestandabdomenandPositronemisisontomography(PET)
areoneforstagingofesophagealcancer.
Stomach,jejunumorcoloncanbeusedforreplacementafter

surgicalremoval

4.Trueregardingacutesialedinitisis/are?
a)Mostcommoninsubmandibularglands
b)Mostcommontypeisviral
c)Canpresentwithstasisofsaliva
d)Theremaybetenderpre-auricularnodes
e)Stoneremovalmaybedonebyprobingthroughoralroute
CorrectAnswer-B:C:E
Answer-B,C,E,Mostcommontypeisviral,Canpresentwith
stasisofsaliva,Stoneremovalmaybedonebyprobing
throughoralroute
Sialadenitisismostcommonintheparotidgland.
Themostcommoncauseofacuteinflammationofthesalivary
glandsismumpsvirus.
Sialadenitisduetobacterialinfectionsismostcommonlycausedby
Staphylococcusaureus.
Stoneorakinkinthesalivaryductcanalsodiminishsaliva
flow/stasisofsaliva.
Surgicalmanagement-
Abscessesrequiredrainage
Glandexcisionincasesofrecurrentacutesialadenitis.

5.Trueregardingchronicpancreatitisis/are?
a)Canpresentwithsteatorrheaandmalabsorption
b)Presentswithmidepigastricpainradiatingtoback
c)Markedlyraisedlevelofamylase&lipase
d)Predisposestocarcinoma
e)Completepancreactomyrelievespaininmajorityofpatients
CorrectAnswer-A:B:D:E
Answer-A,Canpresentwithsteatorrheaandmalabsorption
B,Presentswithmidepigastricpainradiatingtoback
D,PredisposestocarcinomaE,Completepancreactomyrelieves
paininmajorityofpatients
Etiology
Alcoholicpancreatitis-mostcommon
Hyperparathyroidism
Hypertriglyceridemia
Autoimmune
Clinicalfeatures-
Abdominalpainisthemostcommonpresentingsymptom.
Thepatientexperiencesintermittentattacksofseverepain,oftenin
themid-abdomenorleftupperabdomenandoccasionallyraditingin
abandlikefashionorlocatizedtothemidback.
Chronicpancreatitisincludemaldigestion,malabsorption,diarrhea,
bloatingandweightloss.Thismaybedueeithertofearofeatingor
duetopancreaticexocrineinsufficienyandsteatorrhea&azotorrhea
(proteinmalabsorption).
Serumenzymes-
Serumamylaseandlipaselevelsarenormalorslightlyelevatedin

chronicpancreatitis.
Inlaterstageofchronicpancreatitis,atrophyofthepancreatitic
parenchymaresultsinnormalserumenzymelevelsbecauseof
fibrosisofthepancrease.
Treatment-
i)Pancreaticductdrainage
Inpatientswithadilatedpancreaticduct,aRoux-en-Yside-to-side
pancreaticojejunostomyisindicated.
ii)Pancreaticresection-
Ifthediseaseislimitedtotheheadofthepancreas,aWhipple
operation(pancreaticoduodenectomy)canproducegoodresults.

6.q-SOFAscoreincludes?
a)Pulserate
b)Respiratoryrate
c)Systolicbloodpressure
d)Alteredmentation
e)Meanarterialpressure
CorrectAnswer-B:C:D
Answer-B,RespiratoryrateC,Systolicblood
pressureD,Alteredmentation
ItisavalidatedICUmortalitypredictionscore,tohelpidentify
patientswithsuspectedinfectionthatareathighriskforpoor
outcome(definedasin-hospitalmortality,orICUlengthofstay>3
days)outsideoftheICU.
TheqSOFAsimplifiestheSOFAscoredrasticallybyonlyincluding
its3clinicalcriteria.
Itisusedinpatients>l8yearsoldinanon-ICUsetting(pre-hospital,
ward,emergencydepartment,stepdownunit)with
confirmedorsuspectedinfection.
Lowbloodpressure-1
HIGHRESPIRATORYRATE-1
Alteredmentation-1

7.Trueregardinglegulcers&theirlocation
is/are?
a)Arterialinsufficiency-tipofthetoes
b)Arterialinsufficiency-medialsideofleg[abovemedial
malleolus]
c)Venousinsufficiency-abovelateralmalleolus
d)Diabeticneuropathiculcer-planteraspectofmetatarsalhead
e)Pressureulcer-heel
CorrectAnswer-A:C:D:E
Answer-A,Arterialinsufficiency-tipofthetoesC,Venous
insufficiency-abovelateralmalleolusD,Diabeticneuropathic
ulcer-planteraspectofmetatarsalheadE,Pressureulcer-
heel
Venousulcer-
Locatedbelowtheknee,mostoftenontheinnerpart(medial)ofthe
ankles.Thoseassociatedwithlassersaphenousveininsufficiency
mayoccuronouterside(lateral).
Associatedwithaching,swollenlowerlegsthatfeelmore
comfortablewhenelevated.
Arterialulcer-
Usuallyfoundonthefeet,heelsortoes.Toesareafectedmost
commonlyespeciallytips.
Thebordersoftheulcerappearasthoughtheyhavebeen'punched
out'.
Frequentlypainful,particularlyatnightinbedorwhenthelegsareat
restandelevated.

Neurotrophic(diabetic)Ulcers-
Neuropathiculcersusuallyoccurontheplantaraspectofthefoot
underthemetatarsalheadsorontheplantaraspectsofthetoes.
Pressureulcer(pressuresore/decubitusulcer)-
Thehipandbuttockregionsaccountforupto70%ofallpressure
injuries,withischialtuberosity,trochanteric,andsacrallocations
beingmostcommon.
Thelowerextremitiesaccountforanadditionall5-25%ofall
pressureinjuries,withmalleolar,heel,patellar,andpretibial
locationsbeingmostcommon.

8.TRUEstatementregardinghepatocellular
carcinomais/are?
a)FrequentlyassociatedwithraisedAFP
b)Goodprognosisevenwithoutresection
c)Ultrasoundguidedbiopsyisdiagnostic
d)Thereisextensivevascularinvasion
e)Mostcasespresentwithresectabletumor
CorrectAnswer-A:C:D
Answer-A,FrequentlyassociatedwithraisedAFPC,Ultrasound
guidedbiopsyisdiagnosticD,Thereisextensivevascular
invasion
Itisthemostcommonprimarymalignanttumorofliver.
MostimportantriskfactorforHCCisHBVinfection.
Riskfactorsarechronicalcoholism,foodcontaminants
ClinicalFeatures-
Hepatocellularcarcinomashaveastrongpropensityforinvasionof
vascularchannels.Extensiveintrahepaticmetastasisoccursandthe
tumouroccasionallyinvadestheportalyeinorinferiorvenacava.
Unresectedhepatocellularcarcinomahasaverypoorprognosis.
ThefibrolamellarvariantoftheHCCisassociatedwithamore
favorableprognosis.
Diagnosis-
LiverBiopsyunderUSorCTguidance-Thediagnosiscanbe
establishedbypercutaneouscorebiopsyoraspirationbiopsyin
mostpatientsifthebiogsysiteisselected.
MRIistheinvestigationofchoice.

Serologicalmarkers
Elevatedlevelofserumalphafetoproteinsareseenin50-75%
cases.
Treatment-
OnIy15-20%ofHCCareresectablebecauseofmulticentricity,
bilobarinvolvement,portalveininvasionandlymphaticmetastasis.

9.TRUEstatementregardingpyogenicliver
abscessis/are?
a)Morecommononleftsideofliver
b)Surgicaldrainageisthetreatmentofchoice
c)MostcommonorganismresponsibleisE.coli
d)X-raysarediagnostic
e)Diagnosisisconfirmedbyaspirationandculture
CorrectAnswer-C:E
Answer-C,MostcommonorganismresponsibleisE.
coliE,Diagnosisisconfirmedbyaspirationandculture
Itcanbemultiloculatedorasinglecavity.
Itusuallyinvolvestherightlobe(-75%)
Clinicalfeatures
theclassicaldescriptionofhepaticabscessis-fever,jaundiceand
rt.upperquadrantpain;butthisisrarelyseen(-10%cases)
mostcommonpresentationincludesfeverwithchillsand
abdominalpain(FeverisMCsymptom)
Nonspecificsymptomslikemalaiseandanorexiaarealsoseen
Laboratoryinvestigations
AbnormalitiesofLFTsaregenerallyseenbutarenotsevere.
Alkalinephosphatase&transminasesaremildlyelevated
Bilirubiniselevated
UltrasoundandCTarethemaindiagnosticmodalities.
Diagnosisisconfirmedbyaspirationandculture.
(Serologytestsarehelpfulfordiagnosingamoebicabscessnot
pyogenicliverabscess.)

Treatmentinvolvesantibioticsandpercutaneouscatheterdrainage

10.Poorprognosisofbreastcarcinomais
associatedwith?
a)OverexpressionofHer-2/neu
b)Increasedestrogen&progesteronereceptorexpression
c)Triplenegativetumor
d)<2%cellsin'S'phaseofmitosis
e)>65%cellsexpressingKi-67marker
CorrectAnswer-A:C:E
Answer-A,OverexpressionofHer-2/neuC,Triplenegative
tumorE,>65%cellsexpressingKi-67marker
Adiagnosisoftriplenegativebreastcancermeansthatthethree
mostcommontlpesofreceptorsi.e.estrogenreceptors(ER),
Progesteronereceptors(PR),andthehormoneepidermalgrowth
factorreceptor2(HER-2/neugene)arenotpresentinthetumor
cells.
ThistypeofcancerismorecommoninwomenwithBRCA1gene
mutations.

11.LuminalAbreastcancershowsfollowing
feature?
a)Lowgradetumor
b)Her2/neuamplification
c)Goodprognosis
d)Highgradetumor
e)ERnegative
CorrectAnswer-A:C
Answer-A,LowgradetumorC,Goodprognosis
Mostcommonsubtype.
Lowgrade,Slowgrowing.
Bestprognosisofallsubtypes
Hormone-receptor(ERand/orPR)positivewithhighexpression.
HER2negative/lowexpression
LowexpressionoftheproteinKi-67

12.Trueregardingepiduralhematomais/are?
a)Arterialbleed
b)Moredangerousthansubduralvariety
c)OnCTscanitgivesbiconvexlenticularhyperdenseappearance
d)Locatedonlateralsideofhemisphere
e)CommonafterInjuryatpterion
CorrectAnswer-A:B:C:D:E
Answer-A,ArterialbleedB,Moredangerousthansubdural
varietyC,OnCTscanitgivesbiconvexlenticularhyperdense
appearanceD,LocatedonlateralsideofhemisphereE,Common
afterInjuryatpterion
Epiduralhematomabraininjuries(alsoreferredtoasextradural
hemorrhages)involvebloodpoolingbetweentheoutermembrane
(thedura)andtheskull.
Cause-
Injurytomiddlemeningealartery.
Accumulationofblood-
b/wskullanddura
Location-
lateralcerebralconvexities
Clinicalfeatures-
Classicallllucidintervalthencoma,butmorevarialble.
Pupillarydilatationwithcontralateralthenbilaterallimbweakness.
Slowlyevolvingstuporthencoma
Radiologicalfeatures-
Biconvex(lensshapedorlenticular)
InAcutecasesHyperdense(2/3)ormixeddensity(1/3).

InchroniccasesHypodense
Surgicalintervention-
Urgentevacuation

13.Inacuteabdomenexaminationsigns
whichareelicitedinsupineposition
include?

a)Ilio-psoassign
b)Obturatorsign
c)Rovsingsign
d)Carnettsign
e)Balancesign
CorrectAnswer-A:B:C:D
Answer-A,Ilio-psoassignB,ObturatorsignC,Rovsing
signD,Carnettsign
Psoassign(Cope'spsoastestorObraztsova'ssign)-
Itindicatesirritationtotheiliopsoasgroupofhipflexorsinthe
abdomen.
Itiselicitedbyperformingthepsoastestbypassivelyextendingthe
thighofapatientlyingonhissideorsupinewithkneesextended.
Apositivepsoassignontherightmaysuggestappendicitis.
ObturatorSign-
Theobturatorsignisbasedonthesameprincipleasthepsoassign,
thatstretchingapelvicmuscleirritatedbyaninflamedappendix
causespain.
Tostretchtherightobturatorinternusmuscleandelicitthesign,the
patient'srighthipandkneeandtheninternallyrotatestherighthip
withapatientlyingonhissideorsupine.
Rovsing'ssign-
ThepatientissaidtohaveapositiveRovsing'ssignandmayhave

appendicitis.
Itisdoneinsupineposition.
Carnett'ssign-
Carnett'ssignisafindingonclinicalexaminationinwhich(acute)
abdominalpainremainsunchangedorincreaseswhenthemuscles
oftheabdominalwallaretensed.
Asupinepatientcanbeaskedtolifttheheadandshouldersfrom
theexaminationtabletotensetheabdominalmuscles.
Ballancesign-
Thepresenceofadullpercussionnoteinbothflanksofapatient
lyingintheleftdecubitusposition.

14.FALSEstatementregardingurotheilal
bladdertumoris/are?
a)Mostcommonvariety
b)Schistostomiasisisnotariskfactors
c)Stronglyrelatedtosmoking
d)Painisthemostcommonpresentingfeature
e)Mostcommonsiteistrigone
CorrectAnswer-B:D
Answer-B,SchistostomiasisisnotariskfactorsD,Painisthe
mostcommonpresentingfeature
Thereare3typesofbladdercancer-

1. Transitionlcellcancer-(90%)
2. Squamouscellcarcinoma(5-10%)
3. Adenocarcinoma(2%)
Riskfactors-
Cigarettesmoking-it'sthemostcommonetiologicalfactor
Schistostomahaematobium-riskfactorforbothtransitionalcell
carcinoma&SCC
DrugssuchasPhenacetin&chlorphenazine
Clinicalfeatures-
Painlesshaematuriaisthepresentingfeaturein85-90%ofbladder
cancerpatients.
Pain(secondarytoclotretention,tumorextensionintoretro
peritoneumorureteralobstructionormetastasisinbonyPelvis)may
rarelyoccur.
Site-

Trisoneandadjacentpostero-lateralwall

15.TRUEregardinghypertrophicgastric
polypis/are?
a)Mostcommongastricpolyp
b)Pre-malignant
c)Mostlynon-Hodgkin'slymphoma
d)Morecommoninyoungadults
e)Surgeryisdoneiftheyaresymptomatic
CorrectAnswer-A:E
Answer-A,MostcommongastricpolypE,Surgeryisdoneif
theyaresymptomatic
Theyarethemostcommonbenigntumorofstomach.
Gastricpolypsareoffollowingtypes-
1)Hyperplasticpolyp(75%)
Itisthemostcommongastricpolyp.Itisnonneoplastic.Ithasno
malignantpotential.
Hyperplasticpolypsarealsoreferredtoasinflammatorypolyps.
2)Adenomatouspolyp(Adenoma)
Adenomacontainsproliferativedysplasticepitheliumandthereby
hasmalignantpotential.
Treatment-
Gastricpolypsthataresymptomatic>2cmoradenomatous,should
beremoved.

16.Truestatement(s)regardinglymphedema
is/are?
a)Canbecomplicatedbycellulitis
b)CongenitallymphedemaisalsoknownasMillroy'sdisease
c)CommonlycausedbyWouchreriabancrofti
d)Lymphoedemacongenitamorelikelytobeunilateral
e)Lymphedemaprecoxismorecommoninmales
CorrectAnswer-A:B:C
Answer-A,CanbecomplicatedbycellulitisB,Congenital
lymphedemaisalsoknownasMillroy'sdiseaseC,Commonly
causedbyWouchreriabancrofti
Lymphedemaisaninterstitialedemaoflymphaticorigin.
Types-
A.Primarylymphedema-
l)Congenitallymphedema-
Morelikelytobebilateralandinvolvewholeleg.
FamilialversionofcongenitallymphedemaiscalledMilroy'sdisease.
2)Lymphedemaprecox-
Mostcommonformofprimarylymphedema(90%).
FamilialverisonisknownasMeige'sdisease.
B.Secondarylymphedema
Commonorganismscausinglymphaticfilariasisincluding
Wuchereriabancrofti,Brugiamalayi,Brugiatimori.
Themostcommonmanifestationoflymphedemaisedema.
Complications-
Itcanresultinchroniceczema,dermatophytosis,onychomycosis,

cellulitis,lymphangitis,lymphadenitisandinseverecasesskin
ulcers.

17.Stepstakentopreventpost-operative
incisedwoundinfectionis/are?
a)Startantibioticsatleast1daypre-operatively
b)Shavingofhair
c)Onedoseofantibioticjustbeforetheincision
d)Showerpreoperativelyusinganantiseptic
e)Preventintraoperativehypothermia
CorrectAnswer-C:D:E
Answer-C,Onedoseofantibioticjustbeforethe
incisionD,ShowerpreoperativelyusinganantisepticE,Prevent
intraoperativehypothermia
Theyaredefinedasinfectionsthatoccur30daysaftersurgerywith
noimplant,orwithin1yearifanimplantisplacedandinfection
appearstoberelatedtosurgery.
MostSSIsarecausedbyendogenousmicroorganismspresenton
thepatient'sskinwhenthesurgicalincisionismade.
GrampositivebacteriasuchasStaphylococcusaureusarethemost
commoncausativeskin-dwellingmicroorganisms.
Exogenoussourcesofmicroorganismsincludesurgicalinstruments,
operatingroomsurfaces,theair,andpersonnel.
Hairremovalwasoncetheorizedtoreducetheriskofpost-operative
infectionisactuallyassociatedwithincreasedincidenceofSSI.
Preventionstrategies-
Preoperative:

1. Tobaccocessationatleast30dayspriortoelectivesurgery
2. Antimicrobialprophylaxis

AntimicrobialprophylaxisshouldbeadrninisteredonlyforClassI
(cleanwound)andIIwounds(clean/contaminatedwound).Patients
withClassIII(contaminatedwound)orIVwounds(Dirtywound)are
presumedtobetakingantimicrobialtherapyalready.
c)Preoperativeshoweringwithchlorhexidinegluconate4%solution
thenightbeforesurgery.
Appropriatetimingofadministration:
30to6Ominutespriortoincision
1-2hoursforantibioticswithlongerperiodsofinfusion,suchas
vancomycin.
Perioperativetechniques
Preventintraoperativehypothermia
Treatment-
ForsuperficialSSIthisinvolvesopeningthewoundatskinand
subcutaneouslevelandclearingthewound.

18.Trueregardingopsitedressingis/are?
a)Woundcanbeseen
b)Vaporpermeable
c)Impermeabletobacteria
d)Waterpermeable
e)Increasedchancesofmaceration
CorrectAnswer-A:B:C
Answer-A,WoundcanbeseenB,Vapor
permeableC,Impermeabletobacteria
OPSITEpost-opdressing
Itisatransparentadhesivefilm.Thefilmismoisturevapor
permeable.
OPSITEhelpspreventskinmaceration.
OPSITEisresistanttowaterandboilyfluids(waterproof).
ItalsoactsasbarriertobacteriaincludingMRSA.
Allowconstantmonitoringonthewoundandperi-woundareaas
woundcanbeseen.

19.Truestatement(s)regardingspegalian
herniais/are?
a)Protrudesthroughlineaalba
b)Occursattheterminationoftransverseabdominismuscle
c)Occursatthelateraledgeofrectusabdominismuscle
d)Contentsofherniamostlyincludesmallintestine
e)Surgeryisthetreatmentofchoice
CorrectAnswer-C:D:E
Answer-C,Occursatthelateraledgeofrectusabdominis
muscleD,Contentsofherniamostlyincludesmall
intestineE,Surgeryisthetreatmentofchoice
Spigelianherniasoccursecondarytoadefectinthetransversus
abdominismuscleandrectussheathaponeurosis.
Spigelianherniacontentsmostoftenincludessmallintestinebutcan
alsoincludececumappendix,sigmoidcolonoromentum.
Spigelianhernia(orlateralventralhernia)
Aspigelianherniaisaherniathroughthespigelianfasciacloseto
thelevelofthearcuateline.
Spigelianfasciaistheaponeuroticlayerbetweentherectus
abdominismusclemediallyandthesemilunarlinelaterally.
Treatment-
Onceaherniaappears,surgicaltreatmentistheonlywaytorepair
it.

20.TRUEregarding2nddegreesuperficial
burnis/are?
a)Verypainful
b)Dryescharformation
c)Nospontaneoushealing
d)Blisterformation
e)Capillaryrefillingpresent
CorrectAnswer-A:D:E
Answer-A,VerypainfulD,BlisterformationE,Capillaryrefilling
present
Superficial2nddegressburn(Superficialpartialthickness
burn)
Involvesonlyupperdermallayer
Blisterformationoccurs
Erythematous
Blanchontouch
Quitepainful.
Healwithoutscarringin1-2weeks

21.Diagnosticcriteriaforbloodstream
infectionfromcentralvenouscatheter
includesallexcept?

a)Colonycountfromcathetercultureis5timesmorethan
peripheralbloodculture
b)Bloodcultureformcathetershowscolonyformationatleast2
hoursbeforetheperipheralbloodculture
c)Quantitativecathetertipcultureshowing>103CFU/catheter
segmentwherebythesameorganism[speciesandantibiogram]
isisolatedfromthecathetersegmentandaperipheralblood
sample
d)Apparentsourceofbloodstreaminfectionpresent
e)Atleastonepositiveperipheralbloodculture
CorrectAnswer-D
Answer-D.Apparentsourceofbloodstreaminfectionpresent
Itisdefinedasthepresenceofbacteremiaoriginatingfrornan
intravenouscatheter.
Themostcomlnoncauseofnosocomialbacteremia.
Laboratorydiagnosis-
CRBSImeansapatientwithanintravascularcatheterhasatleast
onepositivebloodcultureobtainedfromaperipheral
vein,clinicalmanifestationsofinfections.
Long-termcatheters-
Semi-quantitativegrowthof15cfu/cathetersegmentofthesame
microbefromboththeinsertionsiteculture,andthecatheterhub
culturestronglysuggeststhatthecatheteristhesourceofa

bloodstreaminfection.

22.Apatientwithhistoryoftrauma,
presentedwithmultiplefracture/bilateral
femoralfracture,respiratorydistress&
redurine.Forevaluationofpatientallthe
followingareincludedinmajorcriteriaof
Gurd'scriteriaexcept?

a)Unexplaineddecreaseinplatelets
b)Tachycardia
c)Petechiae
d)CNSdepression
e)Pulmonaryedema
CorrectAnswer-A:B
Answer-A,UnexplaineddecreaseinplateletsB,Tachycardia
Gurd'scriterionisforthediagnosisoffatembolismsyndrome.
GurdandWilson'scriteriaforFES-
Axillary&subconjunctivalpetechiae
Hypxemia
Centralnervoussystemdepression
Pulmonaryedema

23.Inclassificationofcontaminatedwound,
whichofthefollowingareincluded?
a)Resectionofunpreparedbowel
b)Perforatedappendixresection
c)Resectionofintestinalfistula
d)Inguinalherniarepair
e)Hysterectomy
CorrectAnswer-A:B
Answer-A,ResectionofunpreparedbowelB,Perforated
appendixresection
ClassIII(contaminated)-
Open,freshaccidentalwounds.lnaddition,operationswithmajor
breaksinsteriletechnique(e.g.,opencardiacmassage)orgross
spillagefromthegastrointestinaltract,andincisionsinwhichacute,
nonpurulentinflammationisencountered.
Examples:Appendectomy(withperforation/peritonitis)Bowel
Resection(unprepared),bilespillage.

24.Trueaboutcompressionstockingare:
a)Wornevenafterulcerhealstopreventrecurrence
b)Worninmorning&takenoffatnightbeforebedtime
c)Compressionoccursmaximumatcalf
d)Wornonlyatedemasites
e)Providecalfpump
CorrectAnswer-A:B:E
Answer-A,Wornevenafterulcerhealstoprevent
recurrenceB,Worninmorning&takenoffatnightbefore
bedtimeE,Providecalfpump
Thesearespecializedelastichosierydesignedtohelppreventthe
occurrenceofandguardagainstfurtherprogressionof,venous
disorderssuchasedema,phlebitisandthrombosis.
Theyalsoaidinthetreatment&preventionofulcersofthelower
legs.
Compressionstockingsaretightestattheanklesandgradually
becomelessconstrictivetowardthekneesandthighs.Thereforethe
compressionlevelishighestaroundtheankleandlessenstowards
thetopofthehose.
Stockingsarebestapplieduponwakingbeforethepersonhasgot
outofbed.

25.Ulcerativecolitistruestatementis/are?
a)Noskiplesion
b)Almostalwaysincludesrectum
c)Canpresentwithpain&bloodydiarrhea
d)Morecommoninsmokers
e)Transmuralinvolvement
CorrectAnswer-A:B:C
Answer-A,NoskiplesionB,Almostalwaysincludes
rectumC,Canpresentwithpain&bloodydiarrhea


26.Indicationsofhemorrhoidectomyin
hemorrhoidsinclude?
a)Largefirst&seconddegreehemorrhoids
b)Third&fourthdegreehemorrhoids
c)Ifnotabletodifferentiateprolapsedhemorrhoids&lowerrectal
prolapse
d)Complicatedbystrangulations
e)Failureofconservativetherapy
CorrectAnswer-B:D:E
Answer-B,Third&fourthdegreehemorrhoidsD,Complicated
bystrangulationsE,Failureofconservativetherapy
Operativehemorrhoidectomy(excisionalhemorrhoidectomy)-
3rd&4thdegree:hemorrhoids
Otherdegreenotcuresbyconservativemethods
Mixed(combinedinternal/external)hemorrhoids
Hemorrhoidscomplicatedbystrangulation

27.SterileOTzoneis?
a)Changingroom
b)Scrubroom
c)Setuproom
d)Cleanerroomandstores
e)Anaeasthesiainducingroom
CorrectAnswer-B:C:E
Answer-B,ScrubroomC,SetuproomE,Anaeasthesiainducing
room
Zone-3(Sterilearea)
OperatingTheatre
ScrubRoom
AnesthesiaRoom
SetupRoom

28.TrueaboutPhylloidestumoris/are?
a)AssociatedwithBRCA1
b)FNACcandiagnosereliability
c)Treatedwithmastectomy
d)Axillarylymphnodesarecommonlyinvolved
e)AssociatedwithBRCA2
CorrectAnswer-C:E
Answer-C,TreatedwithmastectomyE,AssociatedwithBRCA2
Itisarare,predominantlybenigntumorthatoccursalmost
exclusivelyinthefemalebreast.Phyllodestumoristhemost
commonlyoccurringnonepithelialneoplasmofthebreast.
Largetumor-simplemastectomyisdone

29.Metabolicabnormalityseenincongenital
hypertrophicpyloricstenosisis?
a)Hypochloremichypokalemicmetabolicalkalosis
b)Hyperchloremichypokalemicmetabolicalkalosis
c)Hypochloremichypokalemicmetabolicacidosis
d)Hyperchloremichypokalemicmetabolicacidosis
e)None
CorrectAnswer-A
Answer-A.Hypochloremichypokalemicmetabolicalkalosis
Proiectilenonbillotrsvomitingistheinitialsynptomsofpyloric
stenosis,Thevomitingusuallystartsalter3weeksofage,but
symptomsmaydevelopasearlyasthelstweekoflifeandaslateas
the5monthsoflife.
Hpyochloremichypokalemicmetabolicalkalosis.

30.Congenitalhypertrophicpyloricstenosis
causes
a)Biliousvomiting
b)Nonbiliousvomiting
c)Projectilevomiting
d)Nonprojectilevomiting
e)None
CorrectAnswer-B:C
Answer-B,NonbiliousvomitingC.Projectilevomiting
Thevomitingmayormaynotbeprojectile,howevernon-bilious
vomitingafterfeedisthecharacteristicfeature.
Inabout10-15%ofinfantsvomitusmaycontainblood.Thisbleeding
isduetorefluxesophagitis,acommonassociationwithhypertrophic
pyloricstenosis.
AlsoknowAssociatedanomaliesoccurin6-20%ofpts.This
includesesophagealatresia,Hirshprung'sdisease,anorectal
anomalies,andintestinalmalrotation.

31.Mostcommonpost-operativecomplaint
is/are?
a)Pain
b)Nausea
c)Vomiting
d)Sedation
e)Shivering
CorrectAnswer-A:B:C
Answer-A,PainB,NauseaC,Vomiting
Nauseaandvomitingepisodesstillpersistasthemostcommon
complaint.
Post-operativenauseaandvomiting(PONV)isacommon
complicationofsurgeryandanaesthesia
Sorenessinthethroatifthepatientneedsartificialventilation.
Sorenessandswellingaroundtheincisionsite.

32.Falseaboutabdominalarteryaneurysm
(AAA)is:
a)SurgeryisindicatedwhenAAA>6cm
b)90%ofAAAispresentbelowrenalartery
c)Mortalityrateaftersurgeryis>25%
d)Commonlycausescolonischemia
e)Mostcommoncauseisatherosclerosis
CorrectAnswer-A:C:D
Answer-A,SurgeryisindicatedwhenAAA>6cmC,Mortality
rateaftersurgeryis>25%D,Commonlycausescolonischemia
Itisthemostcommontruearterialaneurysm.
Mostcommonsite(85-90%)isinfrarenal.
Etiology-
Atherosclerosisisthemostcommoncause.
Non-specificaorto-arteritis:-the2ndmostcommoncause.
The5yearriskofruptureofaneurysmof<5cmis1to2%,where
asitis20-40%foraneurysm>5cm.
Investigation
IOCisCECT
Management-
Operativerepairoftheaneurysmwithinsertionofprostheticgraftor
aorticstentgraftisindicatedfor-
ForasymotomaticaneurysmsAAArepairisindicatedifsize>5.5
cms.
InelectivesurgeryofAAAtheoperadvemortalityrateapproximates
aboutl-2%whereasafteracuterupturethemortalityrateof

emergencysurgerygenerallyapproximateabout45-50%.
Complications-
Death(1.8-5%mortalityforelectiveopenrepair<1%for
endovascularrepairand50%iftheAAAhasruptured)
Colonischemiaisl%forelectiverepair15-20%iftheAAAhas
ruptured

33.Hypoparathyroidismcanoccurin:
a)Afterthyroidsurgery
b)Digeorgesyndrome
c)Radicalresectionofhead&neckcancer
d)MENI
e)All
CorrectAnswer-A:B:C
Answer-A,AfterthyroidsurgeryB,Digeorge
syndromeC,Radicalresectionofhead&neckcancer
Geneticdisorders(eg:DeGeorgesyndrome)
.Abnormalparathyroidglanddevelopment
.AbnormalPTHsynthesis
oActivatingmutationsofcalciumsensingreceptor(autosomal
dominanthypocalcemiaorsporadicisolatedhypoparathyroidism)
.Post-surgical(thyroidectomnparathyroidectomnradicalneck
dissection)
oAutoimmunepolyglandularsyndrome

34.Trueregarding10-0suturesis/are?
a)Thickerthan1-0sutures
b)Syntheticsutures
c)Diameteris0.9mm
d)Strongerthan1-0
e)Alloftheabove
CorrectAnswer-B
Answer-B.Syntheticsutures
Thelargerthesizeascribedtothesuture,thesmallerthediameter
be.
10-0sutures-0.2(0.020-0.029)

35.Trueaboutabdominalaorticaneurysm-
a)Mostcommonbelowrenalartery
b)Maypresentsaspulsatileabdominalmass
c)Atherosclerosisismostcommoncause
d)Forasymptomaticaneurysmsrepairisindicatedifthediameter
is>4cm
e)None
CorrectAnswer-A:B:C
Answer-A,B,C,Mostcommonbelowrenalartery,(B)May
presentsaspulsatileabdominalmass(C)Atherosclerosisis
mostcommoncause
9O%ofabdominalaorticaneurysm(AAA)ofsize>4cmindiameter
isduetoatherosclerosis.
Malearemorefrequentlyaffectedthanfemale.
Theaneurysmmostcommonlyarisesbelowthelevelofrenalartery.
Itisusuallydetectedonroutineexaminationasapalpable,pulsatile,
expansile,andnontendermass.

36.Trueaboutpostoperativeileus:
a)Colonrecoversearlierthansmallintestine
b)Smallintestinemostcommonlyaffected
c)Adhesionismostcommoncause
d)Usuallyresolveswithin48-72hour
e)Radiographsshowdiffuselydilatedbowelwithairinthecolon
andrectum
CorrectAnswer-D:E
Answer-D,Usuallyresolveswithin48-72hourE,Radiographs
showdiffuselydilatedbowelwithairinthecolonandrectum
Ileusthatoccursimmediatelyaftersurgeryintheabsenceof
precipitatingfactorsandresolveswlthln2to4daysisreferredtoas
primaryorpostoperativeileus.
Postoperativeileusaffectsthestomachandcolonprimarily.
Afterlaparotomy,smallbowelmotilityretunwithinseveralhours,
gastricmotilitywithin24to48hours,andcolonicmotilityin48to72
hours.
Abdominalradiographsrevealdiffuselydilatedbowelthroughoutthe
intestinaltractwithairinthecolonantdrectum.

37.Trueaboutsecondarybacterial
peritonitis-
a)MarkedleukocytosiswithleftshiftoftheWBCstobandforms
b)Usuallycausedbypolymicrobialinfection
c)Maybeassociatedwithappendicularperforation
d)Associatedwithcirrhosisoftheliver
e)None
CorrectAnswer-A:B:C
Answer-A,Markedleukocytosiswithleft...B,Usuallycausedby
polymicrobial...C,Maybeassociatedwith...
Secondaryperitonitisdevelopswhenbacteriacontaminatethe
peritoneumasaresultofspillagefromanIntraabdominalviscus
Patientsarefebrile,withmarkedleukocytosisandaleftshiftofthe
WBCstobandforms.
Peritonealinfectionisusuallycausedbytwoormorebacterial
strains.

38.Correctstatementaboutintestinal
ischemia:
a)Watershedzonesareusuallyaffected
b)Arterialischemiclesionhasbetterdemarcationthanischemia
causedbyimpairedvenousdrainage
c)Cryptsmaybehyperproliferative
d)Microscopicexaminationmayshowcoagulativenecrosisofthe
muscularispropria
e)Surfaceepitheliumisnormal
CorrectAnswer-A:B:C:D
Answer-AWatershedzones...,B,Arterialischemiclesionhas
better...C,CryptsmaybehyperproliferativeD,Microscopic
examination...
Intestinalresponsestoischemiaoccurintwophases-

1. Theinitialhypoxicinjuryoccursattheonsetofvascular
compromise.
2. Reperfusioninjury,isinitiatedbyrestorationofthebloodsupplyand
itisatthistimethatthegreatestdamageoccurs.
Colonisthemostcommonsiteofgastrointestinalischemia,mucosal
andmuralinfarctionmayinvolveanylevelofthegutfromstomach
toanus.
Therebcoagulativenecrosisofthemuscularispropriawithin2to4
days,andperforationmayoccur.
Microscopicexaminationofischemicintestinedemonstratethe
characteristicatrophyorsloughingofsurfaceepithelium.
Cryptsmaybehyperproliferative.


39.TrueaboutColorectalcancer:
a)Rightsidecancerusuallypresentswithobstructivesymptoms
b)MaybeassociatedwithHNPCC
c)Left-sidedcancerpresentswithalterationinbowelhabit
d)Leftsidecolonismorecommonlyinvolved
e)All
CorrectAnswer-B:C:D
Answer-B,MaybeassociatedwithHNPCCC,Left-sidedcancer
presentswithalterationinbowelhabitD,Leftsidecolonismore
commonlyinvolved
Tumorsoftheleftsideofthecolonwhicharefarmorecommon.lt
usuallypresentwithachangeinbowelhabitorrectal
bleeding,whilemoreproximallesionstypicallypresentlaterwithiron
deficiencyanaemiaoramass.
HereditaryNonpolyposisColonCancer(Lynch'sSyndrome):Itis
characterizedbythedavelopmentofcolorectalcarcinomaatan
earlyage.
Tumorsoftheleftcoloncangraduallyoccludethelumen,causing
changesinbowelhabitswithalternatingconstipationandincreased
frequencyofdefecation.

40.Trueaboutthyroidcancer:
a)Follicularcancerismorecommoninradiationexposedpatient
b)Harthlecellcarcinomahasbetterprognosisthanfollicular
cancer
c)Follicularcancerhavebetterprognosisthanpupillarycancer
d)Insularvariantofpapillarycarcinomahasbetterprognosisthan
papillarycancer
e)MedullarycarcinomaisassociatedwithMEN-2b
CorrectAnswer-E
Answer-E,MedullarycarcinomaisassociatedwithMEN-2b
MedullarythyroidcarcinomainMEN2Bdevelopsearlierandismore
aggresivethaninMEN2A.
PapillaryCarcinomasarethemostcommonformofthyroidCancer.
Harthlecellcarcinomamayhaveaworseclinicalprognosis.

41.TrueaboutSchatzkiring:
a)Containtrueesophagealmuscle
b)Concentricsymmetricnarrowingofloweresophagus
c)Consistsofesophagealmucosaaboveandgastricmucosa
below
d)Dysphagiaisusuallytosolidfoods
e)Associationwithrefluxdisease
CorrectAnswer-B:C:D:E
Answer-B,Concentricsymmetricnarrowingofloweresophagus
CConsistsofesophagealmucosaaboveandgastricmucosa
below,D,DysphagiaisusuallytosolidfoodsE,Associationwith
refluxdisease
Schatzki'sringisathinsubmucosalcircumferentialringinthedistal
oesophagus,usuallyatthesquamocolumnarjunction.
Itconsistsofesophagealmucosaaboveandgastricmucosabelow.
Itdoesnothaveacomponentoftrueesophagealmusclethe
dysphagiaisusuallytosolidfoodsonlyandcomesonabruptlywith
nearlycompleteobstruction.
Thereisastrongassociatianwlthrefluxdisease.

42.Trueaboutintestinaltypeofgastric
cancer-
a)Incidencedecreasednowadays
b)AssociatedwithH.pyloriinfection
c)Morecommoninmalethanfemale
d)Betterprognosisthandiffuse
e)Morecommonthandiffusetype
CorrectAnswer-A:B:C:D
Answer-A,IncidencedecreasednowadaysB,Associatedwith
H.pyloriinfectionC,MorecommoninmalethanfemaleD,Better
prognosisthandiffuse
"Forunclearreason,theincidenceandmortalityratesforgastric
cancerhavedecreasedmarkedlyduringthepast65years.Gastric
cancerincidencehasdecreasedworldwidebutremainshighin
Japan,China,ChileandIreland"-Harrison16/e,p524
Morecommoninwomen.
Theprognosisbelessfavourable.
Intestinalmetaplaslaoccursduetopersistentirritationofthegastric
mucosa,mostcommonlyfromH.pyloriinfection.
TheLaurensystemseparatesgastricadenocarcinomaintointestinal
ordiffusetypesbasedonhistology.

43.Mostcommonpost-operativescomplaints
ofpatientsis/are:
a)Pain
b)Nausea
c)Vomiting
d)Shivering
e)Sedation
CorrectAnswer-A:B:C
Answer-A,B,C,Pain(B)Nausea(C)Vomiting
Nausea,vomitingandpainaremostcommonpost-opcomplications.

44.CorrectstatementaboutIntussusception
inchildrenis/areallexcept-
a)Appendixisthemostcommonleadingpoint
b)Ileocolicisthemostcommonsite
c)Usuallypresentswithpainwithoutvomiting
d)Canbecorrectedbyairandbariumenema
e)Diagnosisisconfirmedonabdominalultrasound
CorrectAnswer-A:C
Answer-A,Appendixisthemostcommonleading
pointC,Usuallypresentswithpainwithoutvomiting
Mostintussusceptionsinchildrenareseenfromtwomonthstotwo
yearsofageintussusceptionissecondarytoapathologicallead
polnt,suchasaMeckel'sdiverticulum,entericduplicationcystor
evenasmallbowellymphoma.
Classically,apreviouslyhealthyinfantpresentswithcolickypainand
vomiting.

45.Non-absorbablesuture(s)is/are:
a)Silk
b)Catgut
c)Polypropylene
d)Polyester
e)Nylon
CorrectAnswer-A:C:D:E
Answer-A,SilkC,PolypropyleneD,PolyesterE,Nylon
Non-absorbablesuturesare:Silk,Linen,Surgical
Steel,Linen,Polyester,Polybutester,Polypropylene,polyethylene,
Nylon.

46.Correctstatementaboutsurgicalsite
infection-
a)CanoccursduetoenvironmentalflorainOT
b)Superficialinfectionrequirere-surgicalexplorationofwound
c)Doesnotoccurlaterthan14dayspost-operative
d)Shavingisbeneficialforprevention
e)Canoccurevenafter1yrincaseofimplantleftinsitu
CorrectAnswer-A:B
Answer-A,Canoccursduetoenvironmentalflorain
OTB,Superficialinfectionrequirere-surgicalexplorationof
wound
TheycanoccuranytimefromOto3Odaysaftertheoperationorup
to7yearafteraprocedurethathasinvolvedtheimplantationofa
foreignmaterial.
Preoperativehairremoval(clipping)shouldbedoneimmeditately
beforeanoPeration.

47.Correctstatementaboutelectrocautery-
a)Inmonopolarmodeoneelectrodeusedatsurgicalfieldand
secondelectrodeisattachedtopatientplate
b)Patientplateshouldhaveconductivejellytoensureproper
contactwithbody
c)Burnmaybeseenatsiteofpatientplate
d)Cuttinghasmorevoltagethancoagulation
e)None
CorrectAnswer-A:B:C:D
Answer-A,B,C,D,Inmonopolarmodeoneelectrodeusedat
surgicalfieldandsecondelectrodeisattachedtopatientplate
(B)Patientplateshouldhaveconductivejellytoensureproper
contactwithbody(C)Burnmaybeseenatsiteofpatientplate
(D)Cuttinghasmorevoltagethancoagulation
High-frequancyalternatingcurrentcanbedeliveredineitherunipolar
orbipolarfashion.
Theunipolar(ormonopolar)deviceiscomposedofagenerator,an
electodeforapplication,andanelectrodeforthereturningcurrentto
completethecircuit.
Complicationsofdiathermy:Electrocution,Explosionanl,Burns.

48.Regardingpre-operativeantibiotic
prophylaxis,whichofthefollowing
statement(s)is/arecorrect:

a)Shouldbestarted2daysbeforesurgery
b)Shouldbegivenhalfanhourpriortosurgery
c)Notneededincleansurgery
d)Shouldbegivenmorethan1hourbeforesurgery
e)None
CorrectAnswer-B
Answer-B.Shouldbegivenhalfanhourpriortosurgery
"Antibioticprophylaxisisindicatedformostcleancontaminatedand
contaminated.
Antibioticsstartedaslateas7to2hoursafterbacterial
contaminationaremarkedlylesseffective.

49.Allarefeature(s)ofBeckwidth-Wideman
syndromeexcept:
a)Omphalocele
b)Macrosomia
c)Hyperglycemia
d)Visceromegaly
e)None
CorrectAnswer-C
Answer-C.Hyperglycemia
Beckwidth-Wiedemannsyndromeisanovergrowthsyndromethatis
characterizedbyvisceromegaly,macroglossia,
macrosomia,mphaloceleandhyperinsulinemichypoglycemia.

50.GradeIII(contaminated)woundis/are-
a)Appendicularperforation
b)Woundcontaminatedwithgrossfecalmaterialspillage
c)Woundinurinarytractwithoutunusualcontamination
d)Surgeryoveracleansite
e)Incisionthroughabscess
CorrectAnswer-B
Answer-B.Woundcontaminatedwithgrossfecalmaterial
spillage
Contaminatedwounds(classIII)includeopenaccidentalwounds
encounteredearlyafterinjury,thosewithextensiveintroductionof
bacteriaintoanormallysterileareaofthebodyduetomajorbreaks
insteriletechnique(e.g.,opencardiacmassage),grossspillageof
viscuscontentssuchasfromtheintestine,orincisionthrough
inflammed,albeitnonpurulenttissue.

51.Feature(s)ofPattersonKellysyndrome:
a)Dysphagia
b)Morecommoninmalethanfemale
c)Achlorhydria
d)Treatmentconsistsofdilationofoesophagusbyoesophageal
bougies
e)All
CorrectAnswer-A:C:D
Answer-A,C,D,Dysphagia(C)Achlorhydria(D)Treatment
consistsofdilationofoesophagusbyoesophagealbougies
Classicalfeaturesofthissyndromeincludedysphagia,irondeficieny
anaemia,glossitis,angularstomatitis,koilonychia(spooningofnails)
andachlorhydria.
Affectsfemalespast40years
Dilatatianofthewebbedareabyoesophagealbougies

52.Allofthefollowingaretrueaboutprostate
specificantigenexcept-
a)Normallevelis10-14ng/ml
b)ItmayElavatedinBPHandCaProstate
c)Itisproducedbyprostate
d)Itisaglycoprotein
e)None
CorrectAnswer-A
Answer-A.Normallevelis10-14ng/ml
Itisaglycoproteinproducedonlyintheprostaticcells(bothbenign
&malignant).
Normalserumlevellessthan4ng/ml
4-10ng/ml>thisrangeiscommonforbothBHPandCa.
Morethan10ng/mlapprox75%willhavecancer.

53.Trueaboutpyomyositis-
a)Staphylococcusaureusisthemostcommoncausative
organism
b)Streptococcuspyogensisthemostcommoncausative
organism
c)Quadricepsisoneofthemostcommonlyaffectedmuscle
d)Treatmentinvolvesdrainageofabscesswithoutantibiotic
coverage
e)BestdetectedbyMRI
CorrectAnswer-A:C:E
Answer-A,C,E,Staphylococcusaureusisthemostcommon
causativeorganism(C)Quadricepsisoneofthemost
commonlyaffectedmuscle(E)BestdetectedbyMRI
Pyomyositisisanacutebacterialinfectionoftheskeletalmuscle
causedbyStaphylococcusaureus.
Itmaybeprimaryorsecondarytoapenetratinginjuryorcontiguous
anatomicinfection.
Ittypicallyaffectsthemajormusclesofthelowerextremityandthe
glutealmuscles.
Patientsusuallypresentswithpain,tendernessandedemaofthe
involvedmusclegroup.
Abscesscandevelopinthemusclegroups.Diagnosisisconfirmed
byneedleaspirationoroperativeincisionanddrainage.
Treatmentinvolvesadequatedrainagealongwithantibiotictherapy
againstStaphylococcusaureus.
Pyomyositisischaracterizedbyalocalizedinfectiousprocess,
conventionalpurulence,lackofsurroundingtissuenecrosis


54.Trueabouttesticularcancer-
a)Seminomaismorecommonthannon-seminoma
b)Cryptorchidismisassociatedwithseminoma
c)RetroperitonealLNdissectionisdoneforgradeIItesticular
tumour
d)Usuallypresentsaspainlesstesticularlump
e)Seminomasmetastasisemainlyviahaematogenousroute
CorrectAnswer-A:B:C:D
Answer-A,Seminomaismore
common...B,Cryptorchidism...C,RetroperitonealLN
dissection...D,Usuallypresents...
Seminomaisthemostcommontypeofgermcelltumour.
Amajorriskforthedevelopmentoftesticularcanceris
cryptorchidism.
Seminomasmetastasisemainlyviathelymphaticsand
haematogenousspreadisuncommon.
Usuallythepatientpresentswithapainlesstesticularlump.

55.Trueaboutobstructivejaundice-
a)Cholangiocarcinomacauseintermittentjaundice
b)Courvoisierlawrelatedtopancreaticheadcancer
c)Cholangitispresentswithfeverandjaundice
d)Increaseinurineurobilinogen
e)Increaseinfecalurobilinogen
CorrectAnswer-B:C
Answer-B,Courvoisierlawrelatedtopancreatichead
cancerC,Cholangitispresentswithfeverandjaundice
examinationoftendemonstratesclinicalsignsofjaundice,cachexia
isoftennoticeableandapalpablegallbladderispresentifthe
obstructionisinthedistalcommnonbileduct(Courvoisier'ssign).
StoneinCBD:Intermitentpain,intermitentfeverandlntermittent
jaundiceareclassicalofstoneinCBD.
Cholangitis:Themostcommonpresentationisfever,epigastricor
rightupperquadrantpain,andjaundiceandisknownasCharcot's
triad.
UrineUrobilinogen:Absent
StoolsSterocobilinogen-Absent

56.Truestatementregardingsurvivingsepsis
guideline:
a)ActivatedproteinCisuseful
b)qS0FAshouldbeassessedincaseofprolongstayofpatientin
ICU
c)SuggestagainsttheuseofIVimmunoglobulinsinpatientswith
sepsis
d)Recommendtheuseoferythropoietinfortreatmentofanemia
associatedwithsepsis
e)Recommendempiricbroad-spectrumtherapy
CorrectAnswer-B:C:E
Answer-B,qS0FAshouldbeassessed...C,Suggestagainstthe
useofIV...E,Recommendempiric...
RecombinanthumanactivatedproteinCwascompletelyomitted
fromthe2Ol2guidelines.
"AhigherSOFAscoreisassociatedwithanincreasedprobabilityof
mortality.
Patientswithsuspectedinfectionwhoarelikelytohaveaprolonged
ICUstayortodieinthehospitalcanbepromptlyidentifiedatthe
bedsidewithqSOFA.
Recommendempiricbroad-spectrumtherapywithoneormore
antimicrobialsforpatientspresentingwithsepsisorsepticshockto
coveralllikelypathogens.
AgainstusingIVhydrocortisonetotreatsepticshockpatientsif
adequatefluidresuscitationandvasopressortherapyareableto
restorehemodynamicstability.

Recommendagainsttheuseoferythropoietinfortreatmentof
anemiaassociatedwithsepsis.

57.Trueaboutmanagementofpepticulcer-
a)Vagotomy-decreasesacidsecretion
b)Earlydumpingsyndromeoccursduetohypoglycaemia
c)Latedumpingconsistsofabdominalandvasomotorsymptoms
d)BillrothIgastrectomyhashighrateofmorbidityandmortality
e)Highlyselectivevagotomyhaslowincidenceofsideeffects
CorrectAnswer-A:D:E
Answer-A,Vagotomy-decreasesacidsecretionD,BillrothI
gastrectomyhashighrateofmorbidityandmortalityE,Highly
selectivevagotomyhaslowincidenceofsideeffects
Highlyselectivevagotomyhasmostsatistactoryoperationfor
duodenalulceration,withalowincidenceofsideeffects.
BillrothIcarrieswithitthemorbidityandmortalityassociatedwith
anygastricresection
Invagotomy,sectionofthevagusnerves,whfuharecritiaally
involvedlnthesecretionofgastricacid.,reducesthemaximalacid
outputbyapprorximately5OPercent.
EarlydumpingconsistsofabdominalandvasomotorsymPtoms.
Latedumpingisreactivehypoglycaemia.Thecarbohydrateloadin
thesmallbowelcausesariseintheplasmaglucose,which
inturn,causesinsulinlevelstorise,causingasecondary
hypoglycemia.

58.Trueaboutsurgicalocclusivedressing-
a)Maintainmoistureinwound
b)Providesmechanicalsupport
c)Preventsmicrobialentry
d)WideMeshishelpful
e)Usedforhighlyexudativewounds
CorrectAnswer-A:B:C
Answer-A,MaintainmoistureinwoundB,Providesmechanical
supportC,Preventsmicrobialentry
Wounddressingscanbecategorizedintofourclasses:
nonadherentfabrics;
absorptivedressings;
occlusive
dressings;
creams,
ointmentsandsolutions
"Occlusivedressingclassprovidesmoistureretention,mechanial
protectionandabarriertobacteria.
Widemeshgauzeiscompositionofabsorptivedressing.

59.Whichofthefollowingnippledischargeis
mostprobablyphysiological-
a)B/Lspontaneousdischarge
b)B/Lmilkydischargewithsqueezingfrommultipleducts
c)U/Lbloodydischarge
d)U/Lbloodydischargewithsqueezingfromasingleduct
e)U/Lspontaneousserousdischarge
CorrectAnswer-B
Answer-B,B/Lmilkydischargewithsqueezingfrommultiple
ducts
Nippledischargeisclassifiedaspathologicifitisspontoneous,
unilateral,blaody,serous,clear,orassociatedwlthamass.
Physiologicaldischargeisusuallybilateral,involvesmultipleducts,
andisassoctatedwithnipplestimulationorbreastcompression.
Unilateral,localizedtoasingleduct.
Themostcommoncauseofspontaneousnippledischargefroma
singleductisasolitaryintraductalpapilloma.

60.ApatienthasGlasgowcomascalescore-
E1VTM2.Whatisnotrequiredforhis/her
management:

a)Headendelevation
b)Totalparenteralnutrition
c)Stressulcerprophylaxis
d)Centrallineplacement
e)None
CorrectAnswer-E
Answer-None
Domain
Response
Score
Spontaneous
4
Tospeech
3
Eyeopenng
Topain
2
None
1
Oriented
5
Confused
4
Bestverbalresponse Inappropriate
3
Incomprehensible
2
None
1
Obeying
5
Localizing
4
Withdrawal
Bestmotorresponse
3
Flexing
4
Extending
1
None

6
5
Deepcomeordeath
4
Totalscore
Fullyalertandoriented 5
3
1

61.TrueaboutBronchogeniccyst:
a)Morecommoninanteriormediastinumthanmiddle
mediastinum
b)Mayhavemalignantpotential
c)Producesymptomswhichareusuallycompressiveinnature
d)Air-fluidmaybeseenonchestX-ray
e)Mostlyasymptomatic
CorrectAnswer-B:C:D:E
Answer-B,C,D,E,Mayhavemalignantpotential(C)Produce
symptomswhichareusuallycompressiveinnature(D)Air-fluid
maybeseenonchestX-ray(E)Mostlyasymptomatic
Bronchogeniccystsarethemostcommonprimarycystsofthe
anteriormediastinum.
ThediagnosisisconfirmedbyCTasasphericalfluid-ormucus
fillednonenhancingmass.Anairfluidlevelmaybepresentonchest
Xray.
Twothirdsofbronchogeniccystsareasymptomatic.
Malignantdegenerationhasbeenreported.

62.TrueaboutFatnecrosisinwomen:
a)Historyoftraumacanbeelicitedin>90%cases
b)Morecommoninlactatingwomen
c)Canbeeasilydifferentiatedfrommalignancyonmammography
d)Nomalignant
e)None
CorrectAnswer-D:E
Answer-D,NomalignantE,None
Traumaispresumedtobethecause,thoughonlyabout5O%of
patientsgiveahistoryofinjury.
Fatnecrosisisararelesionofthebreast.
Acarcinoma,evendisplayingskintetheringandnippleretraction,
andbiopsyisrequiredfordiagnosis.
Fatnecrosiscanmimiccancerbyproducingapalpablemassora
densltyonmamtnographythatmaycontaincalcifications.

63.Correctstatement(s)abouthypospadias
is/are:
a)Urethralopeningonventralaspect
b)Chordaeondorsalaspect
c)Maybeassociatedwithpeniletorsion
d)Proximalvarietiesaremorecommon
e)Couponsspongiosumisdeficient
CorrectAnswer-A:C
Answer-A,C,Urethralopeningonventralaspect(C)Maybe
associatedwithpeniletorsion
Thereisincompletedevelopmentoftheprepuce,calledadorsal
hood,inwhichtheforeskinisonthesidesanddorsalaspectofthe
penileshaftandabsentventrally.
Thereisavariabledegreeofchordae(aventralcurvatureofthe
penismostapparentonerection)
"Peniletorsionisafairlycommoncongenital(presentfrombirth)
conditionthatcanaffectanymaleinfant.

64.Trueaboutomphaloceleisall,EXCEPT:
a)Umbilicalcordinsertsintothesac
b)Incidenceofapproximately1in5000livebirths
c)Abdominalwalldefectmeasures4cmindiameter
d)Visceracoveredbyperitoneum
e)Noneoftheabove
CorrectAnswer-E
Answer-E.Noneoftheabove
Omphalocelereferstoacongenitaldefectoftheabdominalwallin
whichthebowelandsolidvisceraarecoveredbyperitoneumand
amnioticmembrane.
Theumbilicalcordinsertsintothesac.
Theabdominalwalldefectmeasures4cmindiameter.
Omphalocelehasanincidenceofapproximately1in5000live
births.
Theabdominalviscera(commonlyliverandbowel)arecontained
withinasaccomposedofperitoneumandamnionfromwhichthe
umblicalcordarisesattheapexandcenter.

65.Feature(s)ofoesophagealatresiawith
trachea-oesophagealfistula:
a)Droolingofsaliva
b)Associatedwitholigohydramnios
c)Nasogastrictubecannotpassesintostomach
d)MaybeassociatedwithVACTERLanomalies
e)Abdominaldistension
CorrectAnswer-A:C:D:E
Answer-A,C,D,E,Droolingofsaliva(C)Nasogastrictube
cannotpassesintostomach(D)Maybeassociatedwith
VACTERLanomalies(E)Abdominaldistension
Affectedinfantspresentsoonafterbirthwithdroolingandcyanotic
episodesonattemptingtofeed.
Theremayhavebeenpolyhydramniosduetofailuretoswallow
amnioticfluid.
Thediagnosisisconfirmedwhenanasogastrictubegoesnofurther
thantheupperoesophagealpouchonthechestx-rayand
abdominalgassignifiesthetracheo-oesophagealfistula.
TheVACTERLascociation(uertebral,anorectal,cardiac,
tracheoesophageal,renal,andlimbanomalies)ispresentof25%of
cases.
Astheneonatecoughsandcries,airistransmittedthroughthe
fistulaintothestomach,resultinginabdominaldistension.

66.Truestatementregardingsurvivingsepsis
guideline:
a)ActivatedproteinCisuseful
b)qS0FAshouldbeassessedincaseofprolongstayofpatientin
ICU
c)SuggestagainsttheuseofIVimmunoglobulinsinpatientswith
sepsis
d)Recommendtheuseoferythropoietinfortreatmentofanemia
associatedwithsepsis
e)Recommendempiricbroad-spectrumtherapy
CorrectAnswer-B:C:E
Answer:(b)qS0FAshouldbeassessedin...,(c)Suggestagainst
theuseofIV...,(e)Recommendempiricbroad-...
[Ref:Harrison19th/1758;wwwfoamcast.org/2017/01/19;
jamanetwork.com/journakSchwartz9th/128-29]


67.BilateralHilarlymphadenopathyisseenin
-
a)Berylliosis
b)Silicosis
c)Amyloidosis
d)Occupationaldiseases
e)All
CorrectAnswer-A:B:D
Answer-(A)Berylliosis(B)Silicosis(D)Occupationaldiseases
Occupational:Silicosis,coalworker,pneumoconiosis,Berylliosis
TB(unilateralismorecommon)
Castleman'sdisease
Angioimmunoblasticlymphadenopathy
Phenytointherapy

68.Increasedportalvenouspressureisseen
in
a)Buddchairisyndrome
b)Cirrhosis
c)Portalvenousthrombosis
d)IVCobstruction
e)None
CorrectAnswer-A:B:C:D
Answer-A,BuddchairisyndromeB,CirrhosisC,Portalvenous
thrombosisD,IVCobstruction
MCcauseofportalhypertension:Cirrhosis.
Portalhypertensionresultsinsplenomegalywithenlarged,tortuous,
andevenaneurysmalsplenicvessels.
MostbleedingepisodesoccurduringthefirstIto2yearsafter
identificationofvarices.
ColourDoppleristheinvestigationofchoiceforevaluationofPHT.
Splenicpulppressuregivesameasureoftheportalveinpressure.
Posthepatic
Budd-Chiarisyndrome
Inferiorvenacavalwebs
Hepatic-
Sinusoidal
Cirrhosis
Alcoholichepatitis

69.Treatmentofacutepancreatitisinclude(s)
-
a)Intravenousfluid
b)EarlyERCP
c)NSAIDs
d)Nasogastrictube
e)Octeride
CorrectAnswer-A:C:D
Answer-(A)Intravenousfluid(C)NSAIDs(D)Nasogastrictube
Mainlyconservative-fluidresuscitation,analgesics,antibioticsand
anti-emetics
NoroleforTPN(torestthepancreas);fornutritionalsupport?
enteral(nasogastric)feeding
Inpatientswithsevereacutegallstonepancreatitisandsignsof
ongoingbiliaryobstructionandcholangitis,anurgentERCPshould
beperformed
Inpatientswithcholangitis?sphincterotomyorabiliarystent
Indicationsforsurgery:deteriorationdespiteconservativetherapy,
pancreaticabscess/necrosis.

70.Whichofthefollowingsuturematerials
arenotclearedbyproteolyticenzymes
a)Catgut
b)Vicryl
c)Nylon
d)Polyglactin
e)Polyamide
CorrectAnswer-B:C:D:E
Anwer-B,VicrylC,NylonD,PolyglactinE,Polyamide
A)Natural
Silk
Linen
Cotton
B)Synthetic
Nylon
Polypropylene
Polybutest
Braidedpolysters

71.Symptomsofmesentericischemiais/are-
a)Severeabdominalpain
b)Degreeoftendernessisproportionatetoabdominalpain
c)Nauseaandvomiting
d)Diarrhea
e)Absentbowelsound
CorrectAnswer-A:C:D:E
Answer-(A)Severeabdominalpain(C)Nauseaandvomiting
(D)Diarrhea(E)Absentbowelsound
Ischaemiaaffectingthesmallintestinereferredtoasmesentric
ischaemia.
Intestinalischemiacanbeoftwotypes-
Acutemesentericischemia(thecommontype)
Chronicmesentericischemia
C/F
Severeabdominalpain,outofproportiontothedegreeoftenderness
onexarnination,isthehallmarkofacutemesentericischemia.
Associatedsymptomscanincludenausea,vomiting,anddiarrhea.

72.Alongwithsurgicaltreatmentof
glioblastomamultiforme,followingdrugis
approvedforitstreatment

a)Cisplatin
b)Methotrexate
c)Cytarabin
d)Paclitaxel
e)Temozolomide
CorrectAnswer-A:E
Answer-A,CisplatinE,Temozolomide
Carmustine(BCNU)
Cisplatin
Bevacizumab(avastin)

73.Mostcommonsiteofsplenicimplantin
abdomen-
a)Stomach
b)Tailofpancreas
c)Peritonealcavity
d)Suprarenalgland
e)None
CorrectAnswer-C
Answer-C.Peritonealcavity
Hetertopicsplenicimplant(splenosis)mayoccurintheperitoneal
cavity,aftersplenictraumaorsplenectomy.
Splenosisisautotransplantationofsplenictissueafterdisruptionof
spleniccapsulebytraumaorsurgery.
Mostcommonareasofimplantalionareperitoneum,omentumand
mesentry.

74.Whichofthefollowingisnottrue
a)ReefknotisstrongerthanGrannyknot
b)GrannyknotisstrongerthanReefknot
c)Reefknotisappliedfortissueunderpressure
d)ReefknotisstrongerthanSurgeon'sknot
e)Reefknotisbetterthanslipknotforbowelanastomosis
CorrectAnswer-B:D:E
Answer-B,GrannyknotisstrongerthanReefknotD,Reefknot
isstrongerthanSurgeon'sknotE,Reefknotisbetterthanslip
knotforbowelanastomosis
Squareknotisformedbywrappingthesuturearoundtheneedle
holderonceinoppositedirectionbetweenties.
Usually3tiesarerecommended.
Square(Reef)knotismorestablethanGrannyknot-hasless
tendencytoslipwhersubjectedincreasedpressure.
ItmaybeusedtotieSurgicalgut,virginsilk,surgicalcottonand
surgicalstainlesssteel.

75.Trueaboutchestwalltumoris/are
a)Lipomaisthemostcommontumor
b)Fibrousdysplasiaisacommonskeletalsarcoma
c)Desmoidtumorisusuallynotmalignant
d)Chondrosarcomaisthemostcommonmalignanttumor
e)Treatedbysurgicalresectionwithwidesurgicalmargins
CorrectAnswer-B:D:E
Answer-B,Fibrousdysplasia...D,Chondrosarcomaisthe
most...E,Treatedbysurgicalresection...
Morethanhalfofthecestwalltumorsaremalignant.
Themostcommonbenignchestwalltumorsare:
Osteochondroma
Chondroma
Fibrousdysplasia
Chondrosarcomasarethemostcommonprimarychestwall
sarcomaandarisefromtheanteriortractofribsandless
commonlyfromthesternum,scapula,orclavicle.
Themostcommonsoft-tissueprimarymalignanttumorsare
fibrosarcomas.
Mostchestwalltumorsaretreatedwithsurgicalresectionand
reconstruction.

76.Trueaboutlipomais/are-
a)Mostcommonbenigntumorofadults
b)MultiplelipomacanoccurandarecalledasDercum'sdisease
c)Intramuscularlipomamaycausepain
d)Rarelymayoccurinbrain
e)Mayshowslipsign
CorrectAnswer-A:B:C:E
Answer-A,MostcommonbenigntumorofadultsB,Multiple
lipomacanoccurandarecalledasDercum's
diseaseC,IntramuscularlipomamaycausepainE,Mayshow
slipsign
Lipomaisabenigntumorofmatureadiposetissue,
Lipomaisthemostcommonbenigntumorinadults.
Commonsitesaresubcutaneoustissueoverthetrunk,napeofthe
neckandlimbs.
Slipsignispositive:Lipomatendstoslipawayfromtheexamining
fingerongentlepressure.
Presenceofmultiplelipomasisknownaslipomatosisandmultiple
lipomasarecalledDercum'sdisease.
Intramuscularlipomamayinterferewithmusclefunctionandcauses
painonmuscleaction.

77.TrueaboutAbdominalcompartmentis
/are-
a)Intraabdominalpressure>15mmHg
b)Increasedintracranialpressure
c)Decreasedcardiacoutput
d)Hypoventilation
e)Renalfailureandoliguria
CorrectAnswer-B:C:D:E
Answer-(B)Increasedintracranialpressure(C)Decreased
cardiacoutput(D)Hypoventilation(E)Renalfailureandoliguria
ACSisdefinedasanincreaseinintra-abdominalpressure(IAP)
morethan20mmHgassociatedwithneworganfailure/dysfunction.
PhysiologicalConsequencesofIncreasedInfra-abdominal
Pressure-
Decreased-
CardiacOutput
CentralVenousReturn
Visceralbloodflow
Renalbloodflow
Glomerularfilteration
Increased
Cardiacrate
Pulmonarycapillarywedgepressure
Peakinspiratorypressure
Intrapleuralpressure
Hypoventilationandalterationofventilation/perfusiondistribution

leadtohypoxemiaandhypercapnia.

78.Trueregardingmanagementofaxillary
lymphnodesinbreastcarcinoma
a)Alllymph-nodesaredissected
b)Lymphnodedissectionisdoneinmodifiedradicalmastectomy
c)Canbeexploredthroughtheextendedincisionofbreast
d)Completedissectionisrequiredif2sentinellymphnodesare
positive
e)None
CorrectAnswer-B:C
Answer-B,Lymphnodedissectionisdoneinmodifiedradical
mastectomyC,Canbeexploredthroughtheextendedincision
ofbreast
Alllymphnodesarenotdissectedinbreastcarcinoma,speciallyin
stageI&II
Lymphnodesaredissectedinmodifiedradicalmastectomy
Axillacanbeexploredthroughtheverticalincisiontakenforbreast
Completedissectionisnotrequiredof1or2sentinellymphnodes
arepositiveinstageI&II

79.LevelVIlymphnodesinneck-
a)Anteriorcompartment
b)Prelaryngeal
c)Submental
d)Nodesofuppermediastinum
e)Jugularnodes
CorrectAnswer-A:B
Answer-(A)Anteriorcompartment(B)Prelaryngeal
LevelVI
Pretracheal
Paratracheal
Prelaryngeal

80.Inflammatoryodontogeniccystis/are
a)Periapicalcyst
b)Residualcyst
c)Paradentalcyst
d)Eruptioncyst
e)Dentigerouscyst
CorrectAnswer-A:B:C
Answer-A,PeriapicalcystB,ResidualcystC,Paradentalcyst
Residualcyst
Paradentalcyst
Periapicalryst

81.Trueaboutbluntabdominaltrauma-
a)Liveristhemostcommonorganaffected
b)AbdominalpelvicCTscanningishelpfulforevaluatingintra-
abdominalandretroperitonealinjuries
c)>100,000redcells/?Linperitoneallavageisconsidered
positiveandisindicationforexploratorylaparotomy
d)Morethan2m1offreebloodintheabdominalcavityis
indicationforexploratorylaparotomy
e)FAST-USGisusedforinitialevaluation
CorrectAnswer-A:C:E
Answer-(A)Liveristhemostcommonorganaffected(C)>
100,000redcells/?Linperitoneallavageisconsideredpositive
andisindicationforexploratorylaparotomy(E)FAST-USGis
usedforinitialevaluation
Blunttrauma:Inblunttraumatheorgansmostcommonly
injuredarethesolidorgans:
Spleen(MC)
Liver
Kidney
Thepresenceof>100,000redcells/plor>500whitecells/plis
deemedpositive(thisisequivalentto2OmLoffreebloodinthe
abdominalcavity).
PositiveDPLisanindicationforemergencyexploratorylaprotomy.
Hemodynamicallystablepatientssustainingblunttraumaare
adequatelyevaluatedbyabdominalultrasoundorCT.

82.Mostaccuratemethodforthediagnosis
GastroesophagealRefluxDisease(GERD)
is

a)Histologicalstudy
b)Manometry
c)24-hourpHrecordingandelectricalimpedancemeasurement
d)Bariumswallowstudies
e)UpperGIendoscopy
CorrectAnswer-C
Answer-C.24-hourpHrecordingandelectricalimpedance
measurement
ThemostsensitivetestfordiagnosisofGERDis24-hambulatory
pHmonitoring.
Endoscopyisindicatedinpatientswithrefluxsymptomsrefractoryto
antisecretorytherapy;inthosewithalarmingsymptomssuchas
dysphagia,weightloss,orgastrointestinalbleeding;andinthose
withrecurrentdyspepsiaaftertreatmentthatisnotclearlydueto
refluxonclinicalgroundsalone

83.Trueaboutneurogenicclaudication-
a)Lowbackpainispresent
b)Fixedwalkingdistance
c)Painimmediatelyrelievedbyrest
d)Shinyskinoffoot
e)Painrelievedbyleaningforward
CorrectAnswer-A:C:E
Answer-(A)Lowbackpainispresent(C)Painimmediately
relievedbyrest(E)Painrelievedbyleaningforward
Neurogenicclaudicationischaracterizedbylowbackpainradiating
tolowerlimbs(glutealregion,bockofthigh&leg).
Thesesymptomsareespeciallypresentwhenstandinguprightor
walkingandusuallyrelievedwithleaningforwardorsittingdown

84.Allaretrueaboutbilateralbreastcancer
except
a)Bilateralityiscommonwhenthetumorintheprimarybreastis
lobularcarcinoma
b)About5%bilateralcancersaresynchronousand20%bilateral
cancersaremetachronous
c)Morecommoninwomenof>50yearofage
d)BRCAmutationcancersareassociatedhigherprevalenceof
bilateralbreastcancer
e)Usuallyb/1iffamilialbreastcancerpresent
CorrectAnswer-B:C
Answer-B,About5%bilateralcancersaresynchronousand
20%bilateralcancersaremetachronousC,Morecommonin
womenof>50yearofage
BRCAmutationisariskfactorforbilateralbreastcancer.
Riskfactorsforbilateralbreastcancerare-
Youngageatdiagnosis(<50yearsofage).
multicentricdisease
Lobularinvasivecarcinoma.
Radiationexposure
Familialorhereditarybreastcancer.
Bilateralbreastcancerare-
Synchronous(simultaneous)
Metachronous(sequential)
Metachronousbilateralbreastcancerismorecommonthan
synchronous.


85.Trueaboutsurgicalapproachinthyroid
surgery-
a)Incisionismade1cmbelowcricoidcartilage
b)Usuallyverticalincisionisused
c)Thoracicductmaybedamaged
d)Forlobectomythyroidisdissectedattheisthmus
e)Strapmusclesaredividedifgreaterexposureisneeded
CorrectAnswer-A:C:D:E
Answer-(A)Incisionismade1cmbelowcricoidcartilage(C)
Thoracicductmaybedamaged(D)Forlobectomythyroidis
dissectedattheisthmus(E)Strapmusclesaredividedif
greaterexposureisneeded
Kochertransversecollarincision,typically4to5cminlength,is
placedinorparalleltoanaturalskincrease1cmbelowthecricoid
cartilage.
Thesubcutaneoustissuesandplatysmaareincisedsharply.
TheRLNbmostvulnerabletoinjuryinthevicinityoftheligamentof
Bery.
Ifalobectomyistobeperformed,theisthmusisdividedflushtiththe
tracheaonthecontralateralsideandsutureligated.

86.Trueaboutendemicgoiter-
a)Sizemayincreaseduringpregnancy
b)Usuallyhypothyroid
c)Sameasmultinodulargoiter
d)Canturnmalignant
e)None
CorrectAnswer-A:B:D
Answer-(A)Sizemayincreaseduringpregnancy(B)Usually
hypothyroid(D)Canturnmalignant
EndemicGoiteristhepresenceofagoitercausedbynutritional
deficiencyofIodine.
Effectofpregnancy-
Increasedendocrinedemandtheiodinemetabolismischaracterized
bythetendencytodevelopanendogenousiodinedeficiency(ID).
Diffusethyroidglandhypertrophycanbevisiblyobservedasagoiter
oftheneck.
EndemicGoiterisultimatelycausedbyreducedthyroidhormone
levelsitisaccompaniedbytheclinicalsyndromeofhypothyroidism.
Irregular,nodulargoitersduetorepeatedboutsofiodinedeficiency
mayprogresstothyroidfollicularcarcinoma.

87.Onlysimplecholecystectomyisadequate
inwhichstageofgallbladdercancer
a)StageIA
b)StageIB
c)StageIII
d)StageIV
e)None
CorrectAnswer-A
Answer-A.StageIA
AccordingtoTNMstaging,SimplecholecystectomyisdoneforT1a
whichisincludedinstage-I.
T1a-simplecholecystectomy
T1b,II&III-Extendedcholecystectomy
IV-Palliativetreatment

88.Allaretrueaboutbasalcellcarcinoma
EXCEPT:
a)Translucent
b)Retentioncyst
c)Cystofsubmandibulargland
d)Cystofminorsalivarygland
e)All
CorrectAnswer-A
Answer-A.Translucent
Mostcommonsiteisuppereyelid
Usuallyaslow-growing,locallyinvasivemalignanttumourof
pluripotentialepithelialcellsarisingfrombasalepidermisandhair
follicles,henceaffectingthepilosebaceousskin.
Basalcellcarcinomasusuallypresentaspearlypapulescontaining
prominentdilatedsubepidermalbloodvessels(telangiectasias)

89.Trueabouthypertrophicscar-
a)Treatedwithtriamcinolone
b)Resultsfromaprolongedinflammatoryphaseofwoundhealing
c)Mostcommoninpigmentedskin
d)Growbeyondmargin
e)Improvespontaneouslywithtime
CorrectAnswer-A:B:E
Answer-(A)Treatedwithtriamcinolone(B)Resultsfroma
prolongedinflammatoryphaseofwoundhealing(E)Improve
spontaneouslywithtime
Hypertrophicscarsarecharacterizedbyerythematous,pruritic,
raisedfibrouslesionsthattypicallydonotexpandbeyondthe
boundariesoftheinitialinjuryandmayundergopartialspontaneous
resolution.
Itresultsfromaprolongedinflammatorypheseofwoundhealingand
fromunfavourablescarsiting.
Hypertrophicscarsarecommonafterthermalinjuriesandother
injuriesthatinvolvethedeepdermis.
IntralesionalinjectionofTriamcinoloneisalsothet/tofchoicefor
intractablehypertrophicscars.

90.Whichofthefollowingis/aretrueabout
Gastriclymphoma
a)AssociatedwithH.pyloriinfection
b)MajoritybelongstoB-cellHodgkinlymphomas
c)Endoscopicultrasoundisperformedtodeterminethedepthof
gastricwallinvasion
d)Secondmostcommontumorofstomach
e)Diagnosisismadeonbasisofendoscopicbiopsy
CorrectAnswer-A:C:D:E
Answer-A,AssociatedwithH.pyloriinfectionC,Endoscopic
ultrasoundisperformedtodeterminethedepthofgastricwall
invasionD,Secondmostcommontumorof
stomachE,Diagnosisismadeonbasisofendoscopicbiopsy
Thestomachisthemostcommonsiteforextranodallymphoma.
B-celllyphomasofmucosa-associatedlymphoidtissue(MALT
lymphoma).
Lymphomaisthesecondmostcommonprimarycancerofthe
stomach.
Majorityofcases(80%)areassociatedwithchronicgastritisandH.
Pyloriinfection.
Itismostprevalentissixthdecadeoflife.
MALTomasexpressB-cellmarkersCD19andCD20.
Diagnosisismadebyendoscopicbiopsy.
Endoscopicultrasoundisusefultodeterminethedepthofgastric
wallinvasion.
Gastriclymphomasarechemosensitiveandchemotherapyaloneor

alongwithsurgeryisusedforthetreatmentofgastriclymphoma.

91.Feature(s)ofsuperficialpartialthickness
burn(seconddegree)is/are-
a)Hairseasilypluckable
b)Severepain
c)Thrombosedvessel
d)Leatheryskin
e)Blisteringofskin
CorrectAnswer-B:E
Answer-(B)Severepain(E)Blisteringofskin
thesesuperfcialdermalburnsinvolvetheupperlayersofdermis
Blistersareseen
Erythematous
Blanchtotouch
Quitepainful
Healwithoutscarringin1to2weeks


92.Notincludedinsurgicalsafetychecklist
a)Signin
b)Signout
c)Timein
d)Timeout
e)Preanestheticcheckup
CorrectAnswer-C:E
Answer-C,TimeinE,Preanestheticcheckup

93.Allaretrueaboutsuccinate
dehydrogenasedeficientgastrointestinal
stromaltumor(SDH-deficientGIST)
except

a)Morecommoninchildrenandyoungadult
b)NegativeforDOG-1
c)Mostcommonlocationisstomach
d)AssociatedwithCarney-Stratakissyndrome
e)Absentc-kitmutation
CorrectAnswer-B
Answer-B.NegativeforDOG-1
Succinatedehydrogenase(SDH)deficient-GastrointestinalStromal
Tumors(GIST)-
Pathologicalfeatures-
SDHdeficient-GISTsdonothavec-KITmutation-->absenceofc-
KITmutation.
TheystronglyexpressKITandDOG1/Ano-1,butdonothaveKIT
mutation.
Clinicalfeatures-
TheymaYbeassociatedwith:-

1. Carneystratakissyndrome-ParagangliomawithfamilialGIST
2. Carneytriad
Thesetumorshavetendencytoappearinchildrenandyoungadults
Theyoccurexclusivelyinstomach.

94.Whichofthefollowingis/aretrueabout
appendicitis-
a)Bothdiarrheaandconstipationmaypresent
b)Nauseaandvomitingusuallypresent
c)Painoninternalrotationofflexedhip
d)Painonflexionandexternalrotationofhip
e)Initiallypainislocatedintheperiumbilicalregion
CorrectAnswer-A:B:C:E
Answer-A,Bothdiarrheaandconstipationmay
presentB,NauseaandvomitingusuallypresentC,Painon
internalrotationofflexedhipE,Initiallypainislocatedinthe
periumbilicalregion
Clinicalfeatures-
AbdominalPain(mostcommon)isfrequentlynoticedinthe
periumbilicalregion.
Anorexia
Nauseaandvomiting
Diarrheaorconstipation
SignsinAppendicitis-
Rovsingsign
Obturatorsign
Psoassign
Dunphysign
Marklesign
McBurney'ssign

95.Allaretruestatementabouthernias
except
a)Femoralherniahasthehighestriskofstrangulationofgroin
hernias
b)Directinguinalherniacausemoresymptomsthanindirect
c)Directinguinalherniaismostcommoninchildren
d)Indirectinguinalherniaoccurduetopatentprocessusvaginalis
e)Indirectinguinalherniaoccurthroughadefectinfascia
transversalis
CorrectAnswer-B:C
Answer-B,Directinguinalherniacausemoresymptomsthan
indirectC,Directinguinalherniaismostcommoninchildren
Afemoralherniahasthehighestriskofincarcerationand
strangulationofgroinhernias.

96.Differenceb/wfullthicknessandpartial
thicknessgrafts-
a)Partialthicknessgrafthavegoodcosmeticappearance
b)Fullthicknessgraftaregoodforlargearea
c)Edemaundergraftmaycausegraftfailure
d)Thingraftsurvivetransplantationmorereliably
e)Minimalcontractioninfullthicknessgraft
CorrectAnswer-C:D:E
Answer-(C)Edemaundergraftmaycausegraftfailure(D)Thin
graftsurvivetransplantationmorereliably(E)Minimal
contractioninfullthicknessgraft

Typeof
Advantages
Disadvantages
Graft
-Leastresemblesoriginal
skin.
ThinSplit
-BestSurvival
-Leastresistancetotrauma.
Thickness -HealsRaqidly
-PoorSensation
-MaximalSecondary
-Contraction
-Morequalitiesofnarmal
skin.
ThickSplit
-Lowergraftsurvival
-LessContraction
Thickness
-Slowerhealing.
-LooksBetter
-FairSensation
-Mostresemblesnormal
skin.
-Poorestsurvival.
-Donorsitemustbeclosed

-Donorsitemustbeclosed
Full
-MinimalSecondary
surgically.
Thickness contraction
-Resistanttotrauma
-Donorsitesarelimited.
-GoodSensation
-Aestheticallypleasing
Edemaandnecrotictissueundergraftmayhampergraft
acceptance.

97.Mediastinalmass(s)whichis/aremore
commoninposteriormediastinum
a)Lymphoma
b)Thymoma
c)Neurogenictumor
d)Enterogenouscyst
e)Thyroidcarcinoma
CorrectAnswer-C:D
Answer-C,NeurogenictumorD,Enterogenouscyst
MostcommontumorsintheposteriormediastinumareNeurogenic
tumors.
Posteriormediastinalmassesinclude-
Lymphnodeenlargement
Neuroentericcyst(Enterogenouscyst)
Anteriormeningocele

98.Trueaboutundescendedtestis
a)U/LmorecommonthanB/L
b)Missingtestisonpalpationmaybeduetoagenesis
c)10%bilateral
d)Undescendedtestismaybeassociatedwithabsentkidney
e)Stephenfowlertechniqueinvolvesrenalarteryligation
CorrectAnswer-A:B:C:D
Answer-A,U/LmorecommonthanB/LB,Missingtestison
palpationmaybeduetoagenesisC,10%
bilateralD,Undescendedtestismaybeassociatedwithabsent
kidney
Cryptorchidismisthemostcommoncongenitalabnormalityofthe
genitourinarytract.
Cryptorchidismmeanshiddentestis.
Anabsenttestismaybeduetoagenesisoratrophysecondaryto
intrauterinevascularcompromisealsoknownasthe"vanishing
testissyndrome".
Bilaterallyabsenttestesisanorchiawhichis10%cases.
MorecommononRightSide.
Complicationsofundescendedtestes
Torsioncanbeseeninincompletetesticulardescent
Sterilityisseeninbilateralcases(especiallyintra-abdominaltestes)
Incompletetesticulardescentpredisposestomalignantdisease;
cancerismorecommoninanincompletelydescendedtestes-
orchidopexymayormaynotdiminishtherisk.
Atrophyofaninguinaltestesbeforepubertymaypossiblybecaused
byrecurrentminortrauma.


99.Trueabouttorsionoftestisisallexcept
a)Presentswithsuddenpainintestis
b)Commonlyassociatedwithpyuria
c)DopplerU/Sshowsdecreasedbloodflowtothetestis
d)Simultaneousorchipexyoftheothersideshouldalsobedone
e)All
CorrectAnswer-B
Answer-B.Commonlyassociatedwithpyuria
Pyuriaisassociatedwithepidydimo-orchitis,notwithtorsionof
testis.
Torsionisthetwistingofthetestisonthespermaticcord,resultingin
strangulationofthebloodsupplyandinfarctionoftestis.
Itisseencommonlyinadolescents(10-25yrs)
Symptoms-4itpresentsassuddenagonisingpaininthegroinand
thelowerabdomen.Nauseaandvomittingareverycommon.
ColourDopplerdetectsthedecreasedbloodflowtotestisin
torsion

100.TrueaboutMeckel'sdiverticulum-
a)Causesvolvulusofintestine
b)Duetopersistentremnantofthevitellointestinalduct
c)Commonlyfoundonthemesentericsideoftheileum
d)Causesmelena
e)Causeshaemoptysis
CorrectAnswer-A:B:D
Answer-(A)Causesvolvulusofintestine(B)Duetopersistent
remnantofthevitellointestinalduct(D)Causesmelena
Meckel'sdiverticulumisthepersistentproximalpartofthe
vitellointestinalductwhichnormallydisappearsduringintrauterine
life.
BleedinginMeckel'sdiverticulumisusuallytheresultofulcerationin
ilealmucosa
Hemorrhagemaypresentasrectalbleedingormelana.
Volvulusoftheintestinearoundthefibrousbandattachingthe
diverticulumtotheumbilicus.
Mesodiverticularbandisaremnantofleftvitellineartery.

101.Trueabouttuberculosis-
a)Cavitatorylesionsuggestsinactivedisease
b)Rasmussenaneurysmisacomplication
c)Highgradefever
d)Tubercularbronchiectasisoccurinlowerlobes
e)All
CorrectAnswer-B
Answer-B.Rasmussenaneurysmisacomplication
Cavitationisasignofactivedisease,andisconsideredasasignof
reactivation
FeverinTBisusuallylow-gradeandintermittent.
Rasmussen'saneurysmisaninflammatorypseudoaneurysmal
dilatationofabranchofpulmonaryarteryadjacenttoatuberculous
cavityandlifethreateningcomplicationofcavitytuberculosis.
Tuberculosiscausesupperlobebronchiectasis.

102.Condition(s)associatedwithesophageal
carcinoma
a)Achalsia
b)Post-cricoidweb
c)Schatzki'sring
d)Paterson-Kellysyndrome
e)Lyeingestion
CorrectAnswer-A:B:D:E
Answer-A,AchalsiaB,Post-cricoidwebD,Paterson-Kelly
syndromeE,Lyeingestion
rmportantriskfactorsforSCCcarcinomaare:_
i)AlcoholandCigarettesmoking.
1. Mucosaldamagefromphysicalagents-Hottea,Lyeingestion,
Radiationinducedstrictures,Chronicachlasia.
2. otheringestedcarcinogens-Nitrates,Smokedopiates,Fungal
toxinsinpickledvegetables
3. Plummer-vinson-PatersolKellysyndrome(Esophageal(post
cricoid)Web+glossitis+Irondeficiency).
4. Tylosisplamarisetplantaris(congenitalhyperkeratosisandpittingof
palmsandsoles)
5. Dietarydeficienciesofmolybednum,Zinc,VitaminA.
6. Celiacsprue

103.Trueaboutpseudocyst
a)Pancreaticfluidcollection
b)Maybeasymptomatic
c)Well-definedwall
d)Fluidcollectionisalwaysinfectiousinnature
e)Richinpancreaticenzymes
CorrectAnswer-A:B:C:E
Answer-A,PancreaticfluidcollectionB,Maybe
asymptomaticC,Well-definedwallE,Richinpancreatic
enzymes
Pancreaticpseudocystisthemostcommoncomplicationofboth
acuteandchronicpancreatitis.
Itisnotatruecystasitswalldoesnothaveanepitheliallining.
Fluidofthecystisrichinpancteaticamylase.
Mostcommonsiteforpseudopancreaticcystisthebodyandtailof
pancreas.
Clinicalfeatures-
Pancreaticpseudocystsshowawidevarietyofclinicalpresentations
rangingfromcompletelyasymptomaticlesionstomanysymptoms.
AbdominalpainisMCsymptom.
Investigations-
CECTabdomenisinvestigationofchoicefordiagnosisofa
pancreaticpseudocyst.

104.Hyperamylasemiais/areseeninall
except
a)Pancreaticpseudocyst
b)Cysticfibrosis
c)Macroamylasemia
d)Parotitis
e)Chronicpancreatitis
CorrectAnswer-B
Answer-B.Cysticfibrosis
Pancreatitis

1. Acute
2. Chronic
Pancreaticpseudocyst
Pancreaticnecorsis
Pancreatictrauma
Pancreaticcarcinoma
cysticfibrosis

105.Indicationsofsurgeryintuberculosis
include(s)-
a)Streakyhemoptysis
b)Tubercularempyema
c)Persistentbronchopleuralfistula
d)Chroniccough
e)AFB+yebacilliinsputum
CorrectAnswer-B:C
Answer-(B)Tubercularempyema(C)Persistentbronchopleural
fistula
Destroyedlung

1. Persistentbronchopleuralfutula
2. Life-threateninghemoptysis(intractablehemorrhage)
3. Aspergillomainatuberculouscavity
4. Postsurgicalcomplication
5. Performanceofdiagnosticprocedure
6. Tubercularemryerna

106.Raisedintracranialpressure(ICP)in
headinjuryis/aremanagedby
a)Furosemide
b)Mannitol
c)DecompressivecraniectomyishelpfulindecreasingICPbutdo
notaffectneurologicaloutcome
d)Glucosefreefluid
e)Hypotonicsolution
CorrectAnswer-A:B:C:D
Answer-A,FurosemideB,MannitolC,Decompressive
craniectomyishelpfulindecreasingICPbutdonotaffect
neurologicaloutcomeD,Glucosefreefluid
A)Initial
Optimiseelectrolytebalance
Sedation
Seizurecontrol
B)Middle(Intermediate)
Mannitol/furosemide/hyperventilationastemporisingmeasures
Heavysedation
C)Last(Final)
Inductionofthiopentonecoma
Decompressivecraniectomy
HyperglycemiacanaggrevateincreaseICP,thusglucosefreefluid
shouldbeused.
Decompressivecraniectomy-Decompressivecraniectomyisthe
surgicalremovalalargeportionofthecranialvaulttoallow

fortheedematousintracranialcontentstoexpandandsubsequently
reduceICP.

107.Whichofthefollowingistrueabout
suturetechnique:
a)Subcuticularsutureisusedwherecosmeticappearanceis
important
b)Verticalmattresscausescauseprominentstitchmarkscars
c)Horizontalmattressisusedforfragileskin
d)Verticalmattressisusedforfragileskin
e)None
CorrectAnswer-A:B:C
Answer-(A)Subcuticularsutureisusedwherecosmetic
appearanceisimportant(B)Verticalmattresscausescause
prominentstitchmarkscars(C)Horizontalmattressisusedfor
fragileskin
Thehorizontalmattressstitchisasuturetechniqueusedtoclose
wounds.
Thismakesitidealforholdingtogetherfragileskin.
Verticalmattresssuturearethatitprovidesclosureforbothdeep
andsuperficiallayers,andalsoallowsperfecteversionandvertical
oppositionofthesuperficialskinedges.
Subcuticularsuturetechniqueisusedinskinwhereacosmetic
appearanceisimportantarrdwheretheskinedgesmaybe
approximatedeasily.

108.Trueaboutcareofpolytraumapatient:
a)CTscanofwholebodycanbedonetoexcludeinjuries
b)Longspinalcordinjurysupportisgivenifspinalcordinjuryis
suspected
c)Triageofinjurieswithcolourcodinglikered,yellow,green&
blackcanbedoneincaseofmasscasualty
d)Initialfluidmanagementisdoneasearlyaspossible
e)None
CorrectAnswer-A:B:C:D
Answer-(A)CTscanofwholebodycanbedonetoexclude
injuries(B)Longspinalcordinjurysupportisgivenifspinal
cordinjuryissuspected(C)Triageofinjurieswithcolour
codinglikered,yellow,green&blackcanbedoneincaseof
masscasualty(D)Initialfluidmanagementisdoneasearlyas
possible
BecauseofCT,anincreasingamountofbothblunt&penetrating
traumahasbeensafelymanagednonoperatively.
'Theavailabilityofhighresolutionmultislicescanners,promotingthe
developmentofprotocolsinsomecentersthatcallforearly
intergrationofcompletebody(i.e.head,cervicalmspine,chest,
abdomen&pelvis)CTscanningofselectedtraumapatient.
Intravenouslineiscriticalforintravenousvolumeresuscitation.
Triagesysteminvolvesacolor-codingschemeusingred,yellow,
green,white,andblacktags:Redindicateshighprioritytreatmentor
transfer,yellowsignalsmediumpriority,greenindicatesambulatory
patients&blackfordeadormoribund.

109.Trueaboutcentralvenous
catheterization:
a)Cathetermaybeplacedthroughsubclavianvein
b)IftheCVPislowinthepresenceofalowMAPorcardiac
output,thenfluidresuscitationisnecessary
c)Canbeusedforgainingvenousaccessinpatientwithpoor
peripheralveins
d)Subclavianveinrouteissafestintermofcomplications
e)All
CorrectAnswer-A:B:C
Answer-(A)Cathetermaybeplacedthroughsubclavianvein
(B)IftheCVPislowinthepresenceofalowMAPorcardiac
output,thenfluidresuscitationisnecessary(C)Canbeused
forgainingvenousaccessinpatientwithpoorperipheralveins
UsedformonitoringofCVEadministrationoffluidtotreat
hypovolemia&shock,infusionofcausticdrug&totalParenteral
nutrition,asPirationofemboli,insertionoftranscutaneouspacing
leads&gainingvenousaccessinpatientwithpoorperipheralveins.
CVPorrightatrialpressure(RAP)ismonitoredusingacatheter
insertedviaeithertheinternaljugularorthesubclavianvein.
TheCVPmayhelpinassessingtheneedforintravascularfluid
replacemant.

110.Feature(s)of3rddegreeburnis/are:
a)Blisterformation
b)Verypainful
c)Nospontaneoushealing
d)Dryescharformation
e)Thrombosedvesselscanbeseenundertheskin
CorrectAnswer-C:D:E
Answer-C,NospontaneoushealingD,Dryeschar
formationE,Thrombosedvesselscanbeseenundertheskin
Entirethicknessofskindestroyed(intofat)
Anycolor(white,black,red,brown),dry,lesspainful(dermalplexus
ofnervesdestroyed)
Healbycontractionandscardeposition(noepitheliumleftinmiddle
ofwound)
Full-thicknessBurns(3degreeBURN)-thrombosedvesselscanbe
seenundertheskin.

111.TrueaboutBabcockforcep:
a)Teethatend
b)Serrationatend
c)Triangularorificeinblades
d)Usedforsoftdelicatetubularstructure
e)Curvedforcep
CorrectAnswer-C:D
Answer-C,TriangularorificeinbladesD,Usedforsoftdelicate
tubularstructure
Aninstrumentwitharatchet&atriangularerpansionwith
fenestrationsattheoperatingend
Itdoesnothaveanyteeth
Thisinstrumentcanbeusedtoholdintestine,thyroidgland,
mesoappendix,uterinetubeetc.

112.Sacral-teratoma/sacrococcygeal
teratomaisaderivativeof-
a)Primitivestreak
b)Ectoderm
c)Hypoblasts
d)Cranialneuropore
e)None
CorrectAnswer-A
Answer-A.Primitivestreak
Gastrulation&Hensen'snode:
In2ndweek-GastrulationoccursEstablishes3rdgermlayer
(mesoderm).
Gastrulationbeginswithprimitivestreakformationincaudalregion
ofepiblast.
PrimitivenodeorHensen'snodeorprimitiveknot:
Thickeningformedatcranialendofprimitivestreak.
Sacrococcygealformation:
FailureofHensen'snodetoregresscanleadtosacrococcygeal
formation.
Incompleteprimitivestreakregressionleavescaudalremnant
Sacrococcygealteratoma.
Arisefromresidualtototipotentialcellrests(Hensen'snode).
Sacrococcygealteratomademonstratestissuederivedfromallthree
celllayers.
Containvaryingproportionsofmature&immatureelements.

113.Gasusedincreatingpneumoperitonem:
a)Water
b)CO2
c)02
d)Air
e)CO
CorrectAnswer-B:C:D
Answer-B,CO2C,02D,Air
Gasesusedforpneumoperitoneumincludecarbondioxide(CO2),
air,oxygen,nitrousoxide(N20),argon,heliumandmixturesofthese
gases.

114.CO2isusedinlaproscopyforcreating
pneumoperitoneuminplaceofair
because:

a)Lessabsorptionfromperitonealsurface
b)Fastclearancefrombody
c)Lesssolubilityinblood
d)Electrocauteryissafetouse
e)Lessriskofgasembolism
CorrectAnswer-B:D:E
Answer-B,FastclearancefrombodyD,Electrocauteryissafeto
useE,Lessriskofgasembolism
CO2gasinsufflationispreferredbymostlaparoscopistsbecauseit
hasahighdiffusioncoefficientandisanormalmetabolicend
productrapidlyclearedfromthebody.
Also,CO2ishighlysolubleinbloodandtissuesanddoesnot
supportcombustion.
TheriskofgasembolismislowestwithCO2.
CardiacarrhythmiascanoccurwithCO2pneumoperitoneum.

115.Acutehaemorrhagicpancreatitis
producewhichofthefollowingsign:
a)Cullensign
b)Rovsingsign
c)Psoassign
d)GreyTurnersign
e)Kehrsign
CorrectAnswer-A:D
Answer-A,CullensignD,GreyTurnersign
Bleedingintothefascialplanescanproducebluishdiscolourationof
theflanks(GreyTurner'ssign)orumbilicus(Cullen'ssign).

116.Trueaboutlymphangiomacircum
scriptum:
a)Compressibleswelling
b)Oftencontainclearfluid
c)Groupsofvesiclesarefoundonskin
d)Maypresentatbirthorshortlyafterbirth
e)None
CorrectAnswer-B:C:D
Answer-(B)Oftencontainclearfluid(C)Groupsofvesiclesare
foundonskin(D)Maypresentatbirthorshortlyafterbirth
ThistyPePresentsascircumscribedlesionwhichappearsassmall
vaicleorsmallblisterorslightlyelevatedskinpatch
Thesizevariesfrom0.5to4mmindiameter
Alargeareaofskinmaybeinvolvedontheinnersideofthethigh,
buttockontheshoulderorintheaxilla
Thewholelesionissoft&sPongY
Fluctuation,fluidthrill&translucencytestarealwayspositive.The
swellingisnotcompressible
Themarginsoftheswellingareindistinct
Theskinvesiclescontainclearfluid
Itisusuallyprominentatpubertyandmayoftenstartbleeding.

117.Trueregardingurinarycatheterization
with16Frenchsize:
a)Itisthesuitablesizetouseinadultfemale
b)ItcancauseUTIinlongtermuse
c)Usedinmanagementofurethralobstruction
d)Diameteris4mm
e)Steriletechniquemustbeusedforinsertion
CorrectAnswer-A:B:C:E
Answer-A,ItisthesuitablesizetouseinadultfemaleB,Itcan
causeUTIinlongtermuseC,Usedinmanagementofurethral
obstructionE,Steriletechniquemustbeusedforinsertion
Foley'scatheterissterilizedbygammaradiation.UsuallyFoley's
catheteriskeptfor7days.
Size:
Adults--16F
Children--8For10F
Obstructionoftheurethrabyananatomicalconditionthat,makesit
dfficultforonetourinate:prostatehypertrophy,
prostatecancer,ornarrowingoftheurethra.

118.Whichofthefollowingis/aretrueabout
obstructiveuropathy:
a)Strictureinureteriscause
b)Stoneinureteriscause
c)RTAtype4canoccur
d)AbsenceofhydronephrosisonUSGruleoutobstructive
uropathy
e)Whenobstructionisabovethelevelofthebladder,bilateral
hydroureterandhydronephrosisoccurs
CorrectAnswer-A:B:C
Answer-A,StrictureinureteriscauseB,Stoneinureteris
causeC,RTAtype4canoccur
Anyobstructionatordistaltothebladderneckmayleadtoback
pressureaffectingbothkidneys.
Obstructionatorproximaltotheureteralorificeleadstounilateral
damage.
Partialobstructionleadstogradualprogressivemuscular
hypertrophyfollowedbydilation,decompensationand
hydronephroticchanges.
Acquiredurinarytractobstructionmaybeduetoinflammatoryor
traumaticurethralstrictures,ureteralorpelvicstones.

119.WhichareRadio-opaquestones:
a)Pureuricacid
b)Cysteine
c)Phosphate
d)Oxalate
e)Impureuricacid
CorrectAnswer-B:D:E
Answer-(B)Cysteine(D)Oxalate(E)Impureuricacid
RenalCalculus-
Oxalatestones
Phosphatecalculus-struvite
Uricacidanduratecalculi
Cystinecalculus

120.Trueaboutbranchialfistula:
a)Externalopeningneartheanteriorborderof
sternocleido?mastoid
b)Externalopeningatjunctionofupper&middlethirdof
sternocleidomastoidmuscle
c)Developduetopersistenceof2ndbranchialcleft
d)Openinoropharynx
e)Movewithswallowing
CorrectAnswer-A:C:D
Answer-A,Externalopeningneartheanteriorborderof
sternocleido?mastoidC,Developduetopersistenceof
2ndbranchialcleftD,Openinoropharynx
Thisisalwayscongenital&occursduetopersistent2ndbranchial
cleft.
Theexternalorificeisnearlyalwayssituatedinthelowerthirdofthe
neckneartheanteriorborderofthesternocleidomastoid.
Thepatientmaycomplainofadimple,dischargingmucus&the
dimplebecomesmoreobviouswhenthepatientisaskedtoswallow.

121.Trueabout1stdegreehaemorrhoid:
a)Painlessbleeding
b)Malaenaeoccur
c)Constipationmaybepresent
d)Prolapseondefaecation
e)Neverprolapse
CorrectAnswer-A:C:E
Answer-A,PainlessbleedingC,Constipationmaybe
presentE,Neverprolapse
FourDegreesofHaemorrhoids
Firstdegree:bleedonly,noprolapse
Seconddegree:prolapse,butreducespontaneously
Thirddegree:prolapseandhavetobemanuallyreduced
Fourthdegree:permanentlyprolapsed

122.Whichofthefollowingexaminationis
doneinsupineposition:
a)Tredelenburgposition
b)ReverseTredelenburgposition
c)Simsposition
d)Jackknifeposition
e)Lloyd-Daviesposition
CorrectAnswer-A:B:E
Answer-A,TredelenburgpositionB,ReverseTredelenburg
positionE,Lloyd-Daviesposition
"Tredelenburgposition:Lithotomywithheaddown
ReverseTredelenburgposition:Thepatientissupineonthe
operatingtablewiththetabletiltedup15"attheheadendtoreduce
venousengorgement.
"Mostcommonly,thepatientisexaminedintheleftlateral(Sims)
positiorwiththebuttocksoverlyingtheedgeoftheexamination
couchandwiththeaxisofthetorsocrossing,ratherthanparallel
with,theedgeofthecouch.
Thepronejackknifeorknee-elbowpositionsmaybeused.
Lloyd-DaviesPosition:ItisalsoknownastheTrendelenburg
positionwithlegsapart.

123.Aciniccellcarcinomais/arefoundin:
a)Breast
b)Parotid
c)Lacrimalgland
d)Pancrease
e)Minorsalivarygland
CorrectAnswer-B:D:E
Answer-(B)Parotid(D)Pancrease(E)Minorsalivarygland
MajorSalivaryGlands(includingparotid)
MinorSalivaryGlands
Acinarcellcaricinomaaremalignantepithelialneoplasm(of
pancrease)

124.Trueaboutlipcarcinoma:
a)Commoninsmoker
b)Morecommoninlowerlip
c)Morecommoninupperlip
d)Alwayspainfultotouch
e)Distantmetastasisislate
CorrectAnswer-A:B:E
Answer-A,CommoninsmokerB,Morecommoninlower
lipE,Distantmetastasisislate
Lowerlipismoreofteninvolved
lncidenceoforalcancerissixtimesmoreinsmokerthannon-
smoker
Siteofpredilectionisb/wthemidline&commissureofthelip
Mostly,itissquamouscellcarcinoma,oftenseeninmalesintheage
groupof40-70years
L,Nmetastasesdeveloplate.Submental&submandibularnodes
arethefirsttobeinvolved;otherdeepcervicalnodesmayalsoget
involvedlater.

125.Trueaboutmediastinalteratoma:
a)Mostcommontumourofanteriormediastinum
b)Mostcommongermcelltumourofmediastinum
c)Morecommonlyoccursinanteriormediastinum
d)Containteeth,skin,hair,cartilageandbone
e)Histologicallygradedasdifferentiated&undifferentiated
CorrectAnswer-B:C:D:E
Answer-B,Mostcommongermcelltumourof
mediastinumC,Morecommonlyoccursinanterior
mediastinumD,Containteeth,skin,hair,cartilageand
boneE,Histologicallygradedasdifferentiated&
undifferentiated
Mostcommonmalignancyinyoungmenbetweenage15and35
years.
Primarymediastinalgermcelltumors(includingteratomas,
seminomas,andnonseminomatousmalignantgermcelltumors)are
aheterogeneousgroupofbenignandmalignantneoplasms.
Teratomasarethemostcommontypeofmediastinalgermcell
tumors.
Theycontaintwoorthreeembryoniclayersthatmayincludeteeth,
skin,hair(ectodermal),cartilageandbone(mesodermal),or
bronchial,intestinal,orpancreatictissue(endodermal).
MediastinalTeratomasmayeitherbe-
Mature:Welldifferentiated
Immature:Poorlydifferentiated
Withmalignanttransformation

126.Trueaboutinguinalherniainchildren:
a)Occurduetopersistenceofprocessusvaginalis
b)Defectinabdominalmaypresent
c)Mostcommonvarietyisdirecttype
d)Difficulttoreducenon-operatively
e)Morecommoningirls
CorrectAnswer-A:B:D
Answer-A,Occurduetopersistenceofprocessus
vaginalisB,DefectinabdominalmaypresentD,Difficultto
reducenon-operatively
Inchildren,ifinguinal(indirect)herniaispresentinoneside,then
processusvaginalisisintactonotherside.
Malesareaffectedaboutsixtimesmoreoftenthanfemales.
Indirectinguinalhernia-Canoccurinanyagefromchildhoodto
adult.
Inguinalherniaresultsfromafailureofclosureoftheprocessus
vaginalis.

127.Trueregardingtesticulartumour:
a)ForstageIseminoma-orchidectomyaloneusedfortreatment
b)ForstageIseminoma-orchidectomy+radiotherapyusedfor
treatment
c)StageInon-seminomatousgermcelltumour-orchidectmy
+radiotherapyusedfortreatment
d)Stagingisimportantforplanningmanagement
e)i3HCG&AFPmaybeusedasmarkertheresponseof
treatment
CorrectAnswer-B:D:E
Answer-B,ForstageIseminoma-orchidectomy+radiotherapy
usedfortreatmentD,Stagingisimportantforplanning
managementE,i3HCG&AFPmaybeusedasmarkerthe
responseoftreatment
TesticularTumour:Staging
StageI:Thetumourisconfinedtothetestis;
StageII:Nodaldiseaseispresentbutisconfinednodesbelowthe
diaphragm;
StageIII:Nodaldiseaseispresentabovethediaphragm;
StageIV:Nonlymphaticmetastaticdise
Inconfirmedcases,stagingisanessentialstepinplanning
treatment.
HCG&NSGCTswhenraised,thesemarkersareusedtomonitor
theresponsetotreatment.

128.Feature(s)ofFournier'sgangrene:
a)Necrotizingfasciitisofthemalegenitaliaandperineum
b)Diabetesisriskfactor
c)Urgentwidesurgicalexcisionofthedeadandinfectedtissueis
essential
d)Testisneedtoberemovedinallcases
e)None
CorrectAnswer-A:B:C
Answer-A,Necrotizingfasciitisofthemalegenitaliaand
perineumB,DiabetesisriskfactorC,Urgentwidesurgical
excisionofthedeadandinfectedtissueisessential
Fournier'sgangreneisanecrotizingfasctitisofthemalegenitalia
andperineumthatcanberapidlyprogressingandfatalifnottreated
promptly.
Riskfactors-
urethral
strictures,
perirectalabscesses,
poorperinealhygiene,
diabetes,
cancer,
humanimmunodeficiencyvirus(HIV)
Clinicalfeatures-
Clinicalsignsincludefevers,perinealandscrotalpain.
Cellulitis,eschars,necrosis,flakingskin,andcrepitusmayallbe
observed.
Treatment-

Urgentwidesurgicalexcisionofthedeadandinfectedtissueis
essential.

129.Trueaboutparalyticileus:
a)Postoperativeileusprimarilyaffectsthestomachandcolon
b)Mostlyresolveafter24-72hours
c)Returnoffunctionoccursinthefollowingorder:stomach,large
bowelandsmallbowel.
d)Ifopioidisusedaspostoperativeanalgesia,thenchance
increases
e)Isananticipatedcomplicationsofabdominaloperations
CorrectAnswer-A:B:D:E
Answer-(A)Postoperativeileusprimarilyaffectsthestomach
andcolon(B)Mostlyresolveafter24-72hours(D)Ifopioidis
usedaspostoperativeanalgesia,thenchanceincreases(E)Is
ananticipatedcomplicationsofabdominaloperations
Etiology
Postoperative
Adegreeofileususuallyoccursafteranyabdominalprocedure
Self-limiting(24-72hours)
MaybeprolongedinHypoproteinemiaormetabolicabnormality
Intra-abdominalinflammation(peritonitis,abscess,retroperitoneal
hemorrhage)
Reflexileus:followingfracturesofspine/ribs,retroperitoneal
hemorrhageorapplicationofaplasterjacket
Metabolic&electrolytederangements(uremia,hypokalemia,
hyponatremia,hypoandhypomagnesemia,diabeticcoma,
hypoparathyroidism,hypothyroidism)
Drugs(opiates,psychotropicagents,anticholinergicagents,calcium
channelblockers)

Returnoffunctionoftheintestineoccursinthefollowingorder:small
bowel,largebowelandthenstomach
Paralyticileus,alsocalledpseudo-obstruction,isoneofthemajor
causesofintestinalobstructionininfantsandchildren.

130.Whichofthefollowingis/arefeature(s)of
enlargedkidney:
a)Ballotable
b)Upperbordercanbereached
c)Shifttoparacolicgutteroncompression
d)Donotmovewithrespiration
e)Dullonpercussiononrenalangle
CorrectAnswer-A:E
Answer-A,BallotableE,Dullonpercussiononrenalangle
Itisareniformswelling
ltmovesveryslightlyuithrespirationasitcomesdownalittleatthe
heightofinspiration.
ltisballottable
Asickeningsensationisoftenfeltduringmanipulation
Ahandcanbeeasilyinsinuctedb/wtheupperpoleofswelling&the
costalmargin
Percussionwillrewalresonontnoteinfrontofakidneyswellingas
coilsofintestine&colonwillalwaysbeinfrontofthekidney.

131.Incomparisontoulcerativecolitis,which
ofthefollowingis/arefeature(s)of
crohn'sdisease:

a)Transmuralinflammation
b)Morecommoninsmoker
c)MorepANCApositivity
d)TNF-alphainhibitorroleonlyinCD
e)All
CorrectAnswer-A:B:D
Answer-(A)Transmuralinflammation(B)Morecommonin
smoker(D)TNF-alphainhibitorroleonlyinCD
CanaffectanypartofGIT,butmesites?terminalileum,ileocecal
valve,andcaecum.
Etiology-
Smokingisastrongriskfactor
OCPsandAppendicectomyincreaserisk
Morphology-
Skiplesions
Thickbowelwall
Stricturescommon
Transmuralinflammation
Noncaseatinggranulomas
Moderatepseudopolyps
Deep,knife-likeulcers
Fibrosis,serositis-Marked
Investigations-

70%ASCA+ve(antiSaccharomycescerevisiaeAb)10%pANCA
positive
Treatment
Anti-TNFtherapy(Infliximab,adalimumab,certolizumab)-first-line
agentstoinduceremissioninmoderatetoseverediseaseandto
maintainremission

132.Trueaboutbreastcyst:
a)Mostlyseeninpremenopausalperiod
b)Mostcommoninyoungwoman
c)Usuallyunilateral
d)Yellowish-greenishdischarge
e)Adheretounderlyingskin
CorrectAnswer-A:C:D
Answer-A,MostlyseeninpremenopausalperiodC,Usually
unilateralD,Yellowish-greenishdischarge
Classicallyseeninperimenopausalwomenmostlyinlastdecadeof
reproductivelife.
50%ofcystsaremultipleorrecurrent.
Cystfluidcanbestraw,colored,opaque,ordarkgreenandmay
containfleckofdebris.
Diagnosiscanbeconfirmedbyaspirationand/orultrasound.

133.TrueaboutCongenitalhypertrophic
pyloricstenosis:
a)Shorteningofpyloriccanalonbariumcontrastimaging
b)Elongationofpyloriccanalonbariumcontrastimaging
c)Narrowingofpyloriccanalonbariumcontrastimaging
d)ThickenedpyloricmuscleonUSG
e)ChildshouldbegivennormalsalinewithKC1
CorrectAnswer-B:C:D:E
Answer-(B)Elongationofpyloriccanalonbariumcontrast
imaging(C)Narrowingofpyloriccanalonbariumcontrast
imaging(D)ThickenedpyloricmuscleonUSG(E)Childshould
begivennormalsalinewithKC1
Imagingconfirmationissoughtbymostcliniciantodifferentiatefrom
gastroesophagealreflex
Precaution:Emptystomachvianasogastrictubebeforestudy&
Removecontrastatend
Elongation&narrowingofpyloriccanal
Stringsign=passingofsmallbariumstreakthroughelongatedpyloric
channel(mostspecificsign)
Double/tripletracksign:crowdingofmucosalfoldsinpyloricchannel

134.Fibrocysticdiseasedifferfrom
fibroadenomabyhaving:
a)Painshiftwithcycle
b)Freelymobile
c)Usuallybilateral
d)Firm&rubbery
e)Welldemarcatedborder
CorrectAnswer-A:C
Answer-A,PainshiftwithcycleC,Usuallybilateral
Painful,oftenmultiple,usuallybilateralmassesinthebreast.
Rapidfructuationinthesizeofthemassesiscommon.
Frequently,painoccursorincreasesandsizeincreasesduring
premenstrualphaseofcycle.
Theselesionsarealwaysassociatedwithbenignchangesinthe
breastepithelium.
Themicroscopicfindingsoffibrocysticconditionindudecyst(gross
andmicroscopic),papillomatosis,adenosis,fibrosis,andductal
epithelialhyperplasia.
Clinicalfeatures-
Painortendernessoftencallsattentiontothemass.
Dischargefromthenipple.
Discomfortoccursorisincreasedduringthepemenstrualphaseof
thecycle.
Fibroadenoma-
Peakageofincidenceisat20year
Presentwithpainlesslump

Itissmooth,roundbordered,firmtohardinconsistency&freely
mobilewithinthebreast(socalledbreastmouse)

135.Testforincompetentvalvedoneby:
a)Coughimpulsetest
b)Trendelenburg
c)Percussationwavetravelinretrogradedirectionalongvaricose
vein
d)Percussationwavetravelinorthogradedirectionalongvaricose
vein
e)All
CorrectAnswer-A:B:C
Answer-A,Coughimpulse
testB,TrendelenburgC,Percussationwavetravelinretrograde
directionalongvaricosevein
Coughimpulsetest:saphenoofemoralincompetence
TrendelenburgI:saphenofemoralincompetence
TrendelenburgII:Perforatorincompetence
Multipletourniquettest:Siteofperforatorincompetence
Schwartztest:Superficialcolumnofblood

136.Whichofthefollowingis/arefeature(s)of
Keloid:
a)Collagenfibersarerandomlyarranged
b)TypeIVcollagendominates
c)Lesionwithhealthymarginremoved
d)Steroidisveryuseful
e)Goesbeyondmarginofwound
CorrectAnswer-B:D:E
Answer-(B)TypeIVcollagendominates(D)Steroidisvery
useful(E)Goesbeyondmarginofwound
Scarsareoftendescribedasbeingatrophic,hypertrophicand
keloid.
Ahypertrophicscarisdefinedasexcessivescartissuethatdoesnot
extendbeyondtheboundaryoftheoriginalincisionorwound.
Akeloidscarisdefinedasexcessivescartisstethatextendsbeyond
thebounilariesoftheoriginatincisionorwound.
IgEantibodyisleastcommonlyseeninkeloid.
eloidsgrowonparticularsites,theseare?
1. Centralchests(probablymostcommon,notsure)
2. Back
3. Shoulder
4. Earlobes
Etiologyofkeloid-
Keyfactors:Surgeryburns,vaccinations
Elevatedlevelsofgrowthfactor(moreoftype.8,collagen)
Lacerationorabrasion

Overthesternum(incision)
Inheritanceandinjection
Deeppigmentedskin
Histologyofbothhypertrophicandkeloidscars-
Excesscollagenwithhypervascularity
keloidswherethereismoretypeIIIcollagen.
Hypertrophicscarsimprovespontaneouslywithtime,whereaskeloid
scarsdonot.
keloidscarshavethicker,moreabundantcollagenbundles.
Treatment-
Intralesionalinjectionofsteroid(Triamcinoloneacetate)isnow
recommendedasthefirstlineoft/tforkeloid.

137.Allaretrueaboutfullthicknessrectal
prolapseexcept:
a)Elderlyareatrisk
b)Commoninchildren
c)Morecommoninfemale
d)sensationofincompleteevacuation
e)Mayassociatedwithaweakpelvicfloor
CorrectAnswer-B
Answer-B.Commoninchildren
Commonlnelderlywomenwhoaremultipara
ConstiPationisimportantfeature
Tenesmuscommon
Somedegreeofincontinenceoffaeces&flatusisalwayspresent.

138.Trueaboutparamedianincision:
a)Itismadebelowumbilicus
b)Betterinobeseperson
c)Causeabdominalhernia
d)Cutanteriorsurfaceofrectussheath
e)Cutposteriorsurfaceofrectussheath
CorrectAnswer-C:D:E
Answer-(C)Causeabdominalhernia(D)Cutanteriorsurfaceof
rectussheath(E)Cutposteriorsurfaceofrectussheath
Use:provideslateralitytothemidlineincision,allowinglateral
structuressuchasthekidney,adrenalsandspleentobeaccessed.
Location:about2-5cmtotheleftorrightofthemidlineincision.
lncisionisoverthemedialaspectofthetransverseconvexityofthe
rectus.
Layersoftheabdominalwallskin,fascia(camper'sandscarpa's)
andtheanteriorrectussheathareincised.
Incisionsinanteriorandposteriorshectiisseperatedbymuscle
whichactsasabuttress,thereforeclosureandmoresecure.


139.Whichofthefollowingis/aretrueabout
renalcarcinomaofT3astage:
a)InvolveGerotafascia
b)Involveperinephricfat
c)Renalveininvolvement
d)Inferiorvenacavainvolvement
e)Size>10cm
CorrectAnswer-B:C:E
Answer-B,InvolveperinephricfatC,Renalveininvolvement
E,Size>10cm



140.Trueaboutextracorporealshockwave
lithotripsy(ECWL):
a)C/Iinpregnancy
b)Lesssatisfactoryforhardstones
c)Stoneisbreakedintosmallpieceswhichcomesoutwithurine
d)Saferincoagulopathy
e)Endoscopicretrievalissometimesusedforimpactedstone
fragmentproducedbylithotripsy
CorrectAnswer-A:B:C:E
Answer-A,C/IinpregnancyB,Lesssatisfactoryforhard
stonesC,Stoneisbreakedintosmallpieceswhichcomesout
withurineE,Endoscopicretrievalissometimesusedfor
impactedstonefragmentproducedbylithotripsy
Crystallinestonesdisintegrateundertheimpactofshockwaves
producedbytheESWLmachine.
UretericcoliciscommonafterESWL.
TheprincipalcomplicationofESWLisinfection.
"Pregnantwomenand.patientswithlargeabdominalaortic
aneurysmsoruncorrectablebleedingdisordersshouldnotbe
treatedwithESWL.

141.Trueaboutcarcinomagallbladder:
a)Increasedbilirubinleveliscontraindicationforsurgery
b)Palliativetreatmentisforadvanceddisease
c)Metastaticrecurrenceiscommoninliver
d)Whengallbladderwallisinvolved,extendedcholecystectomy
isdone
e)Surgeryshouldbedoneinallcases
CorrectAnswer-B:C:D
Answer-(B)Palliativetreatmentisforadvanceddisease
(C)Metastaticrecurrenceiscommoninliver(D)Whengall
bladderwallisinvolved,extendedcholecystectomyisdone
IfGBcancerisfoundatcholecystectomy&ifmucosaaloneis
involved,thencholecystectomyissufficient
IfGBwallisinvolved,thenextendedcholecystectomyisdone
Radiationhasverysmallbenefit.Chemotherapyalsohasbeentried.
5-FU,mitomycinC,doxorubicinaredrugsused
Prognosisisverypoor.Aggressivesurgery&completeclearance
givebestresults.
Surgeryremainstheonlycurativeoptionforgallbladdercancer.

142.Whichofthefollowingis/aretrueabout
breastselfexamination:
a)Performedindifferentpositionslikestanding,sitting,supineetc
b)Tsurvivalrate
c)Doneabout2-3daybeforemenstrualcycleinpremeno-pausal
women
d)Itshouldbestartonlyafter35year
e)Monthlybreastself-examination(BSE)isrecommended
CorrectAnswer-A
Answer-A.Performedindifferentpositionslikestanding,
sitting,supineetc
Breastself-examination(BSE)hasnotbeenshowntoimprove
survival
TheAmericanCancerSocietynolongerracommendsmonthlyBSE
beginningatage20years.
Premenopausalwomenshouldperformtheexamination7-8days
afterthemenstrualperiod.
Thebreastsshouldbeinspectedinitiallywhilestandingbeforea
mirrorwiththehandsatthesides,overhead,andpressedfirmlyon
thehipstocontractthepectoralismuscles.

143.TrueaboutMallory-Weisstear:
a)Alcoholismisapredisposingfactor
b)Tearinvolvesuptomuscularismucosa
c)Tearinvolvesgastricmucosaneargastro-esophagealjunc-tion
d)Associatedwithhiatushernia
e)ImportantcauseofupperGIbleeding
CorrectAnswer-A:C:D:E
Answer-(A)Alcoholismisapredisposingfactor(C)Tear
involvesgastricmucosaneargastro-esophagealjunc-
tion(D)Associatedwithhiatushernia(E)Importantcauseof
upperGIbleeding
AMallory-Weisstear(MWT)isforcefulorretchingvomitingmay
producealongitudinalmucosaltearimmediatelybelowsquamo
columnarjunctionatthecardiaorgastroesophagealjunctionmainly
andalsoinoesophagus.
ETIOLOGY:
Alcoholism
Hiatushernia
NSAIDabuse
Thetearinvolvesthemucosaandsubmucosabutnotthemuscular
layer

144.Highimperforateanusis/areassociated
with:
a)Underdevelopedpelvic&glutealmusculature
b)Associatedwithmaldevelopedurinarysystem
c)AssociatedwithVACTERLsyndrome
d)Fistulaformationwithbladder
e)Normalanalsphintermechanism
CorrectAnswer-A:B:C:D
Answer-(A)Underdevelopedpelvic&gluteal
musculature(B)Associatedwithmaldevelopedurinary
system(C)AssociatedwithVACTERLsyndrome(D)Fistula
formationwithbladder
Morecommonlythereisfistulatotheurethraorbladderinthemale
ortheuppervaginainthefemale.
Patientswithhighimperforateanusoftenhavedeficientpelvicand
glutealinnervationandmusculature,ahighincidenceofsacral
anomalies.
Anomaliesofthevertebraeandtheurinarytractoccurintwo-thirdof
allpatientswithhighanomalies.
ImperforateanusisassociatedwiththeVACTERLsyndrome.

145.Statementtrueaboutcrushinjury&
syndrome:
a)Oftenseenduringearthquake
b)Myoglobinuriaoccurs
c)Presentsasglomerulonephritis
d)Maycausecompartmentsyndrome
e)Maybeassociatedwithrhabdomyolysis
CorrectAnswer-A:B:D:E
Answer-(A)Oftenseenduringearthquake(B)Myoglobinuria
occurs(D)Maycausecompartmentsyndrome(E)Maybe
associatedwithrhabdomyolysis
Acrushinjuryoccurswhenabodypartissubjectedtoahighdegree
offorceorpressure,usuallyafterbeingsqueezedbetweentwo
heavyorimmobileobjects.
Crushinjuryincludeslacerations,fractures,bleedingbruising
compartmentsyndromeandcrushsyrdrome
Associationbetweencrushinjuryrhabdomyolysisandacuterenal
failureseeninearthquake
Themyoglobinurialeadstorenalfailutefromtubularobstruction.

146.Trueregardingurethralinjury:
a)Straddleinjuriesareoftenassociatedwithposteriorure?thral
injuries
b)Pelvicinjuriesisfrequentlyassociatedwithanteriorure?thral
injuries
c)Probleminvoiding
d)Initialmanagementisinsertionofasuprapubiccatheter
e)Suspectifthereisbloodattheurethralmeatusincaseofpelvic
fracture
CorrectAnswer-A:D:E
Answer-(A)Straddleinjuriesareoftenassociatedwithposterior
ure?thralinjuries(D)Initialmanagementisinsertionofa
suprapubiccatheter(E)Suspectifthereisbloodattheurethral
meatusincaseofpelvicfracture
Urethralinjuriesaremorecommonincasesofbilateralpelvic
injuries.
Diagnosisofurethralinjuryismadebyahighindexofsuspicionin
thepresenceofbloodattheurethralmeatus,inabilitytourinate.
Whenbloodlspresentatthemeatus,retrogradeurethrographyaids
inthediagnosisofurethralinjury.
Urethralinjuriesare-
1.posteriorurethra(abovetheurogenitaldiaphragm)
2.anteriorurethra(belowtheurogenitaldiaphragm)
StentedFoleycatheter-healingoftheurethralinjury.
anteriorurethrainjuriesareassociatedwithstraddleinjuries.

147.Whichofthefollowingisincludedin
managementoftraumaticbraininjury:
a)Mannitolinfusion
b)Decompressivecraniectomy
c)CTscan
d)Pneumococcalvaccination
e)Antiepileptics
CorrectAnswer-A:B:C:D:E
Answer-A,MannitolinfusionB,Decompressive
craniectomyC,CTscanD,Pneumococcal
vaccinationE,Antiepileptics
Initial-
Headup
Loosencollar
Optimiseventilation
Optimiseelectrolytebalance(hypertonicfluidsifnecessory),treat
hyperglycemia
Sedation
Seizurecontrol
Intermediate
(Mannito/furosemide/hyperventilationstemporisingmeasures)
Heavysedation
Paralysis
CSFdrainagebyinsertionofEVD
Cooling
Final

Inductionofthiopentonecoma
Decompressivecraniectoy

148.Videoassistedthoracotomyis/are
commonlyusedfor:
a)Pneumonectomy
b)Lungbiopsy
c)Hemothoraxwithactivebleeding
d)Repairoflargevesseltearinthorax
e)Diagnosisandstagingofthoracicmalignancies
CorrectAnswer-A:B:C:E
Answer-A,PneumonectomyB,LungbiopsyC,Hemothoraxwith
activebleedingE,Diagnosisandstagingofthoracic
malignancies
Thoracoscopyorvideo-assistedthoracoscopicsurgery:
Pneumonectomy,lobectomyandempyemadrainageareall
possible.
Lungbiopsyandthetreatmentof-recurentpneumothoraxarethe,
mostfrequentindications.
Thotacotomyisrequiredformanagementofinjuriestothelungs,
heart,pericardium,andgreatvessel.

149.Whichofthefollowingis/aretrue
regardingparathyroidglandsurgeryin
parathyroidadenoma/hyperplasia:

a)SuperiorparathyroidglandliesposteriortoRLN
b)Glandcanbedifferentiatedfromsurroundingtissueduetoits
colour
c)Thepresenceofanormalparathyroidglandatoperation
indicatesthatthetumorremovedisanadenomaratherthan
parathyroidhyperplasia
d)IntraoperativePTHestimationisdonetocheckstatusofgland
removal
e)6%personhave5parathyroidgland
CorrectAnswer-A:B:C:D
Answer-A,Superiorparathyroidglandliesposteriorto
RLNB,Glandcanbedifferentiatedfromsurroundingtissuedue
toitscolourC,Thepresenceofanormalparathyroidglandat
operationindicatesthatthetumorremovedisanadenoma
ratherthanparathyroidhyperplasiaD,IntraoperativePTH
estimationisdonetocheckstatusofglandremoval
Thereareusuallyfourparathyroidglands,whichlieontheposterior
surfaceofthethyroid.
Commonsitesforectopicparathyroidsarethethyrothymicligament,
superiorthyroidpoles,tracheoesophagealgroove,retroesophageal
space,andcarotidsheath.

150.Whichofthefollowingis/arefeatureof
highlyselectivevagotomyincomparison
totruncalvagotomywithdrainage:

a)Bettertolerated
b)Dumpingismorecommon
c)Diarrheaisless
d)Highrecurrencerate
e)Operativemortalityrateveryless
CorrectAnswer-A:C:D:E
Answer-A,BettertoleratedC,DiarrheaislessD,Highrecurrence
rateE,Operativemortalityrateveryless
Highlyselectivevagotomy(HSV),alsocalledparietalcellvagotomy
orproximalgastricvagotomy.
Itpreservesthevagalinnervationtotheantrumandpylorus.
Gastricemptyingofsolidsistypicallynormalinpatientsafterparietal
cellvagotomy.
TherecurrencerateishigherwithHSVthanwithvagotomyand
antrectomy.
ThemostpopulardrainageprocedureistheHeineke-Mikulicz
pyloroplasty.

151.Trueaboutesophagealvarices:
a)Leftgastricveinisportalvesselinvolved
b)Occuratmidesophaguslevel
c)Occuratpharyngeallevel
d)Epigastricveinissystemicveininvolved
e)Occuratlowerendofesophagus
CorrectAnswer-A:E
Answer-A,LeftgastricveinisportalvesselinvolvedE,Occurat
lowerendofesophagus
Thepharynxextendsfromthebaseoftheskulldowntotheinferior
borderofthecricoidcartilage(aroundtheC6vertebrallevel),where
itbecomescontinuouswiththeesophagus.

152.TrueaboutCaroli'sdisease:
a)Intrahepaticbileductdilation
b)Jaundicemaybeseen
c)1stSerumalkalinephosphatase
d)Notassociatedwithportalhypertension
e)Surgeryistreatmentofchoicelocalizedhepaticinvolvement
CorrectAnswer-A:B:C:E
Answer-A,IntrahepaticbileductdilationB,Jaundicemaybe
seenC,1stSerumalkalinephosphataseE,Surgeryistreatment
ofchoicelocalizedhepaticinvolvement
Congenital,multiple,irregulardilatationsofintrahepaticductswith
stenoticsegmentsinbetween
Complications:intrahepaticstoneformation,bililarysepsis
Associatedwithcongenitalhepaticfibrosisandmedullarysponge
kidney
Premalignant
Treatment:
Firstlinetherapy:biliarydrainagewithERCPandPTC
Diffuse:Livertransplantation
Localized:Segmentalresection

153.Allaretrueaboutmedullarythyroid
carcinomaexcept:
a)Involvestheparafollicularcell
b)Radiosensitive
c)Amyloidstromaispresent
d)Elevatedlevelofcalcitonin
e)Highlevelofcarcinoembryonicantigen
CorrectAnswer-B
Answer-B.Radiosensitive
Thesearetumoursoftheparafollicular(Ccells)derivedfromthe
neuralcrest.
Thereisacharacteristicamyloidstroma.
Highlevelsofserumcalcitoninandcarcinoembryonicantigenare
producedmedullarytumours.
Calcitoninlevelsfallefterresectionandriseagainwithrecurrence
makingitavaluabletumourmarkerinthefollowupofpatientswith
thisdisease.
Diarrhoeadueto5-hydroxytrptamineorprostaglandins.
TumoursarenotTSHdependentanddonottakeupradioactive
iodine.

154.Whichofthefollowingis/aretrueabout
breastcarcinomainmaleexcept:
a)Oftenpresentedatadvancedstageatthetimeofdiagnosis
b)Associatedwithgynaecomastia
c)Stagingisdifferentthanfemale
d)Tamoxifeneisusedintreatment
e)Associatedwithexcessendogenousorexogenousoestrogen
CorrectAnswer-C
Answer-C.Stagingisdifferentthanfemale
Thefollowingconditionshavebeenreportedtobeassociatedwith
anincreasedriskofbreastcancerinmen:chronicliverdisorders,
suchascirrhosis,chronicalcoholism,schistosomiasis;ahistoryof
mumpsorchitis,undescendedtestes,ortesticularinjury;and
feminization,geneticallyorbyenvironmentalexposure.Incontrast,
gynecomastiaalonedoesnotappeartobeariskfactor

155.FullformofSCIWORAis:
a)Spinalcordinjurywithradiographicabnormality
b)Spinalcordinjurywithradiographicaberration
c)Spinalcordinjurywithoutradiographicaberration
d)Spinalcordinjurywithoutradiographicabnormality
e)Spinalcordinjurywithvertebralfracturewithradiographic
abnormality
CorrectAnswer-D
Answer-D.Spinalcordinjurywithoutradiographicabnormality
SCIWORA(SpinalCordInjuryWithoutRadiographicAbnormality)
wasfirstdevelopedand,introducedbyPangandWilbergerwho
usedittodefine'clinicalsmyptomsoftraumaticmyelopathywithno
radiographicorcomputedtomographicfeatutesofspinalfractureor
instability.

156.Whichofthefollowingisthefeature(s)
ofposteriorcranialfossafracture:
a)Bleedingfromear
b)Discolourationofskin&collectionofbloodoccurintheregion
ofmastoidprocess
c)Boggyswellinginthenapeoftheneck
d)Bleedingfromnose
e)CSFrhinorrhoea
CorrectAnswer-B:C
Answer-(B)Discolourationofskin&collectionofbloodoccur
intheregionofmastoidprocess(C)Boggyswellinginthenape
oftheneck
Extravasationofbloodinthesuboccipitalregioncausingboggy
swellinginthenepeoftheneck.
9th,10th&llthcranialnervesmaybeinyolved
Battlesign:Discolourationofsiin&collectionofbloodoccurinthe
regionofmastoidprocess.

157.Symptomofoveractivebladderare:
a)Increaseddaytimefrequency
b)Nocturia
c)Hesitancy
d)Dysuria
e)Urgency
CorrectAnswer-B:E
Answer-(B)Nocturia(E)Urgency
Symptomsofoveractivebladder(OAB)resultfromsuddenand
involuntarycontractionsofthebladdermuscles,leadingto-
Suddenneedtourinate(urinaryurgency)and/or
Urinaryincontinence(leakageofurine)
urinaryfrequency(havingtourinateoften)&
Nocturia(urinatingfrequentlyatnight)

158.TrueaboutPsoasabscess:
a)Hipextensionincreasespain
b)Staphylococcusismostcommoncause
c)Presentedwithbackpain
d)TBcancause
e)Causesreferredpaintothehip&groin
CorrectAnswer-A:C:D:E
Answer-(A)Hipextensionincreasespain(C)Presentedwith
backpain(D)TBcancause(E)Causesreferredpaintothehip
&groin
Psoasabscessesmaybeprimaryorsecondary
Primarypsoasabscesses,whichoccurwithoutassociateddisease
ofotherorgans,arecausedbyhematogenousspreadof
Staphylococcusaureus.
ThemostcommoncauseisCrohn'sdisease.
Mycobacteriumtuberculosisasthemajorcausativeorganism.

159.Trueaboutincisionalhernia:
a)Incidenceisabout10-15%ofallabdominalwallhernia
b)Lesschanceinobeseperson
c)Morecommoninwoman
d)Commonlycausedbylowerabdominalsurgery
e)Mayoccurduetoimproperhealingofabdominalincision
CorrectAnswer-A:C:D:E
Answer-(A)Incidenceisabout10-15%ofallabdominalwall
hernia(C)Morecommoninwoman(D)Commonlycausedby
lowerabdominalsurgery(E)Mayoccurduetoimproperhealing
ofabdominalincision
Thesearisethroughadefectinthemusculofasciallayersofthe
abdominalwallintheregionofapostoperativescar.
Incisionalherniasaretwiceasconnnoninwomenasinmen.
"Incisionalherniasaccountforl5%to20%ofallabdominalwall
hernias.
Etiology-
Poorsurgicaltechnique.Inadequatefascialbites,tensiononthe
fascialedges.
Age
Obesity
ascites,
pregnancy,

160.Whichoffollowingis/aretruein
accordancetorevisedgastriccarcinoma
staging:

a)Allgastrictumourswhoseepicentreiswithin5cmofthe
gastrooesophagealjunctionandextendintotheoesophagus
arenowclassifiedaccordingtotherevisedgastricstaging
b)5Nodeinvolvement-N2
c)T1N2M0-StageII
d)PeritonealspreadisMO
e)AnytumourthatperforatestheserosaisnowclassifiedasT4
disease
CorrectAnswer-B:C:E
Answer-(B)5Nodeinvolvement-N2(C)T1N2M0-StageII
(E)Anytumourthatperforatestheserosaisnowclassifiedas
T4disease

Stage TNM
Features
0
Ti,NOMO Nodenegative;limitedtomucosa
Nodenegative;invasionoflamina
IA
T1NOMO propriaorsubmucosa
T2NOMO Nodenegative;invasionofmuscularispropria
IB
T1N1M0
T1N2M0
Nodepositive;invasionbeyondmucosabutwithin
II
T2N1M0
wall
T3NOMO Nodenegative;extensionthroughwall
T2N2M0
Nodepositive;invasionofmuscularispropriaor
IIIA
T3N1-2M0 throughwall

T3N1-2M0 throughwall
IIIB
T4N0-1M0 Nodenegative;adherencetosurroundingtissue
IIIC
T4N2M0
Nodepositive;adherencetosurroundingtissue
T1-4N0-2
Distantmetastases
M1

161.Dumpingsyndromeischaracterizedby
allofthefollowingexcept:
a)Colic
b)Tremorsandgiddiness
c)Hyperglycemia
d)Epigastricfullness
e)None
CorrectAnswer-C
Answer-C.Hyperglycemia
Dumpingsyndromeisarelativelyraredisorderinwhichthestomach
contentsaredeliveredtooquicklytothesmallintestine.
Itoccursasaphysiologicalreactiontotheconsumptionoftoomuch
simpleorrefinedsugarinsomepersons,whensimplesugarexits
thestomachtoorapidlyitattractsfluidintotheupperintestine,and
thebloodvolumedecreasesasitattemptstoabsorbthesugar.
Thevasomotorsymptomscomprisegeneralweakness,pallor,
sweating,palpitationandlightheadedness.
symptomsofgastrointestinaldisturbancesuchasepigastric
discomfort,nausea,vomitingandpossiblyanepisodeofdiarrheais
observed.
Thebiochemicalchangesthatoccurindumpingsyndromeare
hyperinsulinaemiafollowedbyhypoglycaemia

162.Whichofthefollowingis/aretrue
Boerhaavesyndrome:
a)Occurduetoincreaseinthoracicesophagealpressure
b)Morecommononleftlateralwalloftheesophagus
c)Causedbyseverevomiting
d)Causeshudro-pnemothoraxascomplication
e)Maybemisdiagnosedasmyocardialinfarction
CorrectAnswer-A:B:C:D:E
Answer-(A)Occurduetoincreaseinthoracicesophageal
pressure(B)Morecommononleftlateralwalloftheesophagus
(C)Causedbyseverevomiting(D)Causeshudro-pnemothorax
ascomplication(E)Maybemisdiagnosedasmyocardial
infarction
Boerhaave'ssyndromeiswhenapersonvomitsagainstaclosed
glottiscausingfullthicknessoesophagealrupture.
Pressureinoesophagusresultsinburstinginthelowerthirdwhichis
commonestsite.
Clinicalfeatures-
Severechestpain
Abdominalpain
Subcutaneousemphysema
Shock
Hanmann'ssign-crunchingeffectofchest
Mackel'striad-vomiting,chestpain&subcutaneousemphysema
MostperforationsarefoundabovetheGEJontheleftlateralwallof
theesophagus.

Thepressureintheesophagusrapidlyincreases&itburstsatits
weakcstpointinthelowerthird
MCsiteofperforationisatthemidthoracicesophagusontheright
sideattheleveloftheazygosvein.
Maybemisdiagnosedasmyocardialinfarctionorasperforated
Pepticulcerorpancreatitis.

163.FeaturesofReidelthyroiditisare:
a)Presentsasagoitre
b)Painful
c)AssociatedwithIgG4relatedsystemicdiseases
d)Fibrosisofinterstitialthyroidstroma
e)Presentwithhypothyroidism
CorrectAnswer-A:C:D
Answer-(A)Presentsasagoitre(C)AssociatedwithIgG4
relatedsystemicdiseases(D)Fibrosisofinterstitialthyroid
stroma
Itisararedisorderthattypicallyoccursinmiddle-agedwomen.
Itpresentswithaninsidious,painlessgoiterwilhlocalsymptomsdue
tocompressionoftheesophagus,trachea,neckveins,orrecurrent
laryngealnerves.
Densefibrosisdisruptsnormalglandarchitecture.
Thyroiddysfunctionisuncommon.
Treatmentisdirectedtosurgicalreliefofcompressivesymptoms.
Tamoxifenmayalsobebeneficial

164.TrueaboutSickeuthyroidsyndrome:
a)NormalTSH
b)T4toT3conversionimpaired
c)HighTSH
d)T4high
e)IncreasedreverseT3
CorrectAnswer-A:B:E
Answer-(A)NormalTSH(B)T4toT3conversionimpaired
(E)IncreasedreverseT3
Themostcommonhormonepatterninsickeuthyroidsyndrome
(SES)isadecreaseintotalendunboundT3levekl(lowT3
syndtome)withnormallevelsofT4aadTSH.

165.Allaretrueaboutsalivaryglandtumor
except:
a)Parotidglandismostcommonsiteofinvolvement
b)Warthintumouralmostalwaysfoundintheparotidgland
c)Minorglandtumoursaremostlymalignant
d)Parotidtumoursaremostlymalignant
e)Superficialparotidectomyisdoneinpleomorphicadenoma
CorrectAnswer-D
Answer-D.Parotidtumoursaremostlymalignant
"Warthin'stumorarisesalmostexclusivelyintheparotidgland(the
onlytumorvirtuallyrestrictedtotheparotid)"
Theparotidglandisthemostcommonsiteforsalivarytumours.
Tumoursinvolvingfhesublingualglandareextremelyrareandare
usually.
Tumoursofthesubmandibularglandareuncommonandusually
presentasaslow-growingpainlessswellingwithinthe
submandibulartriangle

166.Preferredshuntprocedureinpatientwith
portalhypertensionhavingacceptable
operativeriskandadequateliver
functionis-

a)Endtosideportocavalshunt
b)Endtoendportocavalshunt
c)Mesocavalshunt
d)Distalsplenorenalshunt
e)None
CorrectAnswer-D
Answer-D.Distalsplenorenalshunt
Portosystemicshuntproceduressuchassplenorenalshuntand
mesocavalshunt,maybeindicatedinpatientswithcomplicationof
portalhypertension.
Giventheearlyocclusionrateandtheneedforconstant
surveillance,itisgenerallyadvisedthatTIPSshouldbereservedfor
ChildCclassificationofcirrhosis,whereasadistalsplenorenalshunt
issafe,durable,preferredandeffectivetreatmentinpatientswith
acceptableoperativeriskandstillgoodliverfunction.

167.Trueaboutcleftlip:
a)Occursd/tdefectinfusionoffrontal&nasalprocess
b)Onlybilateralcasesareassociatedwithcleftpalate
c)Repairedinneonatalperiod
d)Unilateralcasesaremorecommoninleftside
e)All
CorrectAnswer-D
Answer-D.Unilateralcasesaremorecommoninleftside
Amidlinecleftlipispresentwhenthereisfailureoffusionbetween
Medialnasalprocesses.
Leftsideunilateralcleftiscommon.
CommonesttypeofcleftlipisCombinedwithcleftpalate.
Cleftingofthelipand/orpalateisfelttooccuraroundtheeighth
weekofembryogenesis,eitherbyfailureoffusionofthemedial
nasalprocessandthemaxillaryprominenceorbyfailureof
mesodermalmigrationandpenetrationbetweentheepithelialbilayer
oftheface.
Repair-
Ruleoftens:Forincreasedanestheticsafety,aninfantshould
1. Be10weeksold.
2. Weigh10pounds.
3. Haveahemoglobinlevelofatleast10mg/dL

168.Whichofthefollowingis/aretrue
regardingmanagementofatrauma
presentingwithshock:

a)TransfusionofPCV:FPP:Plateletshouldbein1:1:1ratio
b)Firsthemodynamicstabilizethepatient,thengoforCTscan
c)FirstgoforCTscan,thenstabilizethepatient
d)CVPlineshouldbeplaced
e)None
CorrectAnswer-B:D
Answer-(B)Firsthemodynamicstabilizethepatient,thengofor
CTscan(D)CVPlineshouldbeplaced
Hypovolemicshockisthemostcommontypeofshockseenin
traumapatient&occurssecondarytoacutebloodloss.
Thesubclaviantbinternaljugularveinshouldbereservedforthose
patientsinwhommajorvenousintra-abdominalinjuriesorpelvic
fracturespreventeffectiveuseoffemoralapproach.
Resuscitationconsistsofaninitialbolusof2Lofcrystalloidsolution.

169.Allaretrueaboutacuteappendicitis
except:
a)ClinicallyindifferentfromMeckeldiverticuluminchildren
b)Lymphoidhyperplasiamaycausesacuteappendicitis
c)Painshifttorightiliacfossa
d)UltrasoundismorediagnosticthanCTscan
e)None
CorrectAnswer-D
Answer-D.UltrasoundismorediagnosticthanCTscan
Painistheearliestfeature,whichisfrequentlyfirstnoticedatthe
periumbilicalregion
SoonthepainshiftstotheRIFandchangesincharacter
E.coliisthemostcommonorganism;enterococciismostcommon.
Riskfactorsforperforationofappendix
Immunosuppression
Diabetesmellitus
Fecolithobstruction
Pelvicappendix
Previousabdominalsurgery
Contrast-EnhancedCT(CECT)?investigationofchoicespeciallyin
unclassicalcases
UltrasoundismorediagnosticthanCTscan

170.Whichofthefollowingistrueabout
esophagealadenocarcinoma:
a)Obesityisariskfactor
b)Mostcommoninmiddle&lower1/3
c)IncidenceinBarrett'soesophagus
d)Squamousmetaplasiaisariskfactor
e)Chronicgastroesophagealrefluxanetiologyofadenocar-
cinoma
CorrectAnswer-A:C:E
Answer-(A)Obesityisariskfactor(C)IncidenceinBarrett's
oesophagus(E)Chronicgastroesophagealrefluxanetiologyof
adenocar-cinoma
Riskfactorsforesophagealadenocarcinoma(Current
Gastroenterology)
Barretsesophagus
GastroesophagealRefluxDisease(GERD)
Scleroderma
Smoking
Alcohol
Historyofcoloncancer
MedicationsLongtermuse(>5years)ofTheophylline&agonists.
Themostcommonsiteofadenocarcinomaesophagusisthelower
1/3rdofesophagus

171.Trueaboutintestinalpseudoobstruction:
a)Maybecausedbyhypokalemia
b)Bezoarscancausepseudo-obstruction
c)Neostigmineusedintreatment
d)Maybeassociatedwithdiverticulosis
e)Colonoscopyiscontraindicated
CorrectAnswer-A:C
Answer-(A)Maybecausedbyhypokalemia(C)Neostigmine
usedintreatment
Intestinalpseudoobstructioniscausedbythefollowing

1. Disordersofthenervoussystem(Familialautonomicdysfunction,
Hirschsprungdisease,Chagasdisease)
2. Diseasesaffectingmusclesandnerves(Musculardystrophy,SLE,
Ehlers-Danlossyndrome,hypokalemia)
3. Disordersoftheendocrinesystem(DM,Hypothyroidism,
Hyperparathyroidism),and
4. Medication(Narcotics,Laxatives,Tricyclicantidepressants,
Phenothiazines).Ogilive'ssyndromeisacuteintestinalpseudo
obstruction.

172.Trueaboutsigmoidvolvulus:
a)Mostcommonspontaneoustypeinadult
b)Lessfiberdietisariskfactor
c)Treatmentincluderesuscitation&decompression
d)Mostcommontypeofcolonicvolvulus
e)Lowrecurrenceafterconservativemanagement
CorrectAnswer-A:C:D
Answer-(A)Mostcommonspontaneoustypeinadult
(C)Treatmentincluderesuscitation&decompression(D)Most
commontypeofcolonicvolvulus
Mostcommonsiteofvolvulus
Rotationinsigmoidvolvulusnearlyalwaysoccurinanticlockwise
direction
PREDISPOSINGFACTOR:
Age
Institutionalizedorneurologicallyimpairedorpsychiatricpatients
Bandofadhesion
LongpelvicmesocolonwithNarrowattachment
Theinitialmanagementisresuscitationfollowedbyendoscopic
decompressionanddetorsion.

173.Trueaboutdiverticulardiseaseofcolon:
a)Rightsidemorecommon
b)Sign&symptomsindistinguishablefromirritablebowel
syndrome
c)Profuse&painlessbleeding
d)Sigmoidismostcommonsite
e)None
CorrectAnswer-B:C:D
Answer-(B)Sign&symptomsindistinguishablefromirritable
bowelsyndrome(C)Profuse&painlessbleeding(D)Sigmoidis
mostcommonsite
Diverticulamostcommonlyaffectthesigmoidcolon.
Diverticulaaremostoftenasymptomatic(diverticulosis).
Presentclinicallywithsepsisorhaemorrhage.
ComplicationsofDiverticularDisease-
Painandinflammation(Diverticulitis).
Perforation
Intestinalobstruction
Haemorrhage
Fistulaformation
Hemorhagefromcolonicdiverticulaistypicallypainless&profuse.

174.Whichofthefollowingistrueabout
primarylymphoedema:
a)Maybecongenital
b)Lymphoedemacongenitamorecommonlyoccurbilaterally
c)Conditionimproveswithmassage
d)Lymphatichyperplasia
e)All
CorrectAnswer-A:B:C
Answer-(A)Maybecongenital(B)Lymphoedemacongenita
morecommonlyoccurbilaterally(C)Conditionimproveswith
massage
Itisduetoaninheritedabnormalityofthelymphaticsystem,
sometimestermed'congenitallymphaticdysplasia'.
Primarylymphoedemaismuchmorecommoninthelegsthanthe
arms.
Lymphoedemacongenitaismorecommoninmales.
Lymphoedemapraecox(onsetfrom2to35years)isthreetimes
morecommoninfemales

175.Allaretrueaboutthymomaexcept:
a)MCtumorinanteriormediastinum
b)Treatmentisthymectomy
c)Symptomaticcasespresentasendocrineabnormalities
d)Maybeassociatedwithmyastheniagravis
e)Associatedwiththyroiditis
CorrectAnswer-C
Answer-C.Symptomaticcasespresentasendocrine
abnormalities
CharachteristicSyndromes
MyaestheniaGravis(mostcommon)
AcquiredHypogammaglobulinemia(10%)
PureRedCellAplasiac

176.Trueaboutinvasivethymoma:
a)Benigninnature
b)MaybeassociatedwithEBV
c)Malignantinnature
d)Epithelialcellsaremostcommonlyofthecorticalvariety
e)None
CorrectAnswer-A:B:D
Answer-(A)Benigninnature(B)MaybeassociatedwithEBV
(D)Epithelialcellsaremostcommonlyofthecorticalvariety
Invasivethymomareferstoatumorthatiscytologically,benignbut
locallyinvasive.
Thesetumorsaremuchmotelileclytometastasize.
Theepithelialcellsaremostcommonlyofthecorticalvariety,with
abundantcytoplasmandroundedvesicularnuclei,andareusually
mixedwithnumerousthymocytes.
Bydefinition,invasivethymomasPenetratethroughthecapsuleinto
surroundingstructures.

177.Trueaboutcystosarcomaphylloides:
a)Usuallybilateral
b)Usuallyoccurinfemaleoverageof40
c)Notinvolvenipple-areolacomplex
d)Treatmentismastectomyofmalignantlesions
e)Allarebenigninnature
CorrectAnswer-B:C:D
Answer-(B)Usuallyoccurinfemaleoverageof40(C)Not
involvenipple-areolacomplex(D)Treatmentismastectomyof
malignantlesions
Itisamalignanttumour
Itoftenmetastasisestoaxillarynodes
Itistreatedbyradicalmastectomy
Usuallyoccurinwomenovertheageold4Oyears.

178.Trueaboutanorectalabscess:
a)Ischiorectalismostcommontype
b)Primarymodalityoftreatmentisantibioticwithoutdrain
c)Rupturecancausefistulaformation
d)Commonindiabetics
e)Drainageofpuswithantibioticsismainstayoftreatment
CorrectAnswer-C:D:E
Answer-(C)Rupturecancausefistulaformation(D)Commonin
diabetics(E)Drainageofpuswithantibioticsismainstayof
treatment
Therearefourtypes-abscess-perianal(mostcommon),ischiorectal,
submucous&pelvirectal.
Anorectalabscesscanruptureinsideaswellasoutsideresultingin
afistula.
Treatmentisdrainageofpusinfirstinstancetogetherwith
appropriateantibiotics.

179.Whichofthefollowingindicatepoor
prognosisinbothRanson&Glasgow
scaleofacutepancreatitis:

a)Albumin
b)Alanineaminotransferase
c)Aspartateaminotransferase
d)Lactatedehydrogenase
e)Basedeficit
CorrectAnswer-A:B:C:D
Answer-(A)Albumin(B)Alanineaminotransferase(C)Aspart
ateaminotransferase(D)Lactatedehydrogenase
Earlypredictorsofseverityat48hoursincluded3Ranson'ssigns
andAPACHEIIscore8.
Arecentscoringsystemfortheearlypredictionofmortalitywas
developedinacutepancreatitis.Thisscoringsystemknownasthe
BedsideIndexofSeverityinAcutePancreatitis(BISAP),
incorporatesfiveclinicalandlaboratoryparametersobtainedwithin
thefirst24hoursofhospitalization.BUN>25,Impairedmental
status,SIRS,Age>60years,Pleuraleffusiononradiography.
Presenceofthreeormoreofthesefactorswasassociatedwith
increasedriskforin-hospitalmortality.
ModifiedGlasgow/PANCREASscore
PaO2<8kPa(60mmhg)
Age>55years
Neutrophils:WBC>15?109/l

Calcium<2mmol/l
Renalfunction:(Urea>16mmol/l
Enzymes:(AST/ALT>200iu/LorLDH>600iu/L)
Albumin<32g/l
Sugar:(Glucose>10mmol/L)

180.Apatienthash/ohypertension&
diabetes.Themechanismofhigher
chanceofulcerinthispatientisdueto:

a)Hyperglycemiassuppressimmunity
b)Vascularinsufficiency
c)Atheroscleroticchanges
d)Venousstasis
e)Tropicskinchanges
CorrectAnswer-A:B:C:E
Answer-(A)Hyperglycemiassuppressimmunity(B)Vascular
insufficiency(C)Atheroscleroticchanges(E)Tropicskin
changes
Trophicchangefromperipheralneuropathy,ischaemiaasaresultof
atheroma,andlowresistancetoinfectionbecauseofexcesssugar
intheTissue.
Neuropathy,abnormalfootbiomechanics,peripheralarterialdisease
(PAD),andpoorwoundhealing.

181.GlasgowcomascaleE3M4V5represents:
a)Spontaneouseyeopening
b)Eyeopeningwithpainstimulus
c)Incomprehensiblesound
d)Orientedconversation
e)Decorticateposturing
CorrectAnswer-D
Answer-(D)Orientedconversation
Domain
Response
Score
Spontaneous
4
Tospeech
3
Eyeopening
Topain
2
None
1
Oriented
5
Confused
4
Bestverbalresponse Inappropriate
3
Incomprehensible
2
None
1
Obeying
6
Localizing
5
Withdrawal
4
Bestmotorresponse Flexing
3
Extending
2
None
1
3
Deepcomaordeath
Totalscore
15
Fullyalertandoriented

Fullyalertandoriented

182.Featureof3degreeburn:
a)Painpresent
b)Transudationoffluidpresent
c)Wholedermisdestroyed
d)Erythematousinappearance
e)Blisterformed
CorrectAnswer-C
Answer-C.Wholedermisdestroyed
Thewholeofthedermisisdestroyedoftheburns.
Clinically,theyhaveahard,leatheryfeel,Theappearancecanvary
fromthatsimilartothepatient'snormalskintocharredblack
dependingupontheintensityoftheheat..Thereisnocapillayreturn
after,thrombosedvesselscanbeseenundertheskin.
Theseburnsarecompletelyanaesthetised:aneedlecanbestuck
deepintothedermiswithoutanypainorbleeding.

183.Complication(s)ofSalivarygland
operation:
a)Subcutaneousfistula
b)Oppositeglandcompensatoryhypertrophy
c)Gustatorysweating
d)FacialNervedamage
e)All
CorrectAnswer-A:C:D
Answer-(A)Subcutaneousfistula(C)Gustatorysweating
(D)FacialNervedamage
ComplicationsofParotidGlandSurgery-
Temporaryfacialnerveweakness
Transectionofthefacidnerveandpermanentfacialweakness
Sialocele
Facialnumbness
Permanentnumbnessoftheearlobeassociatedwithgreatauricular
nervetransection
Frey'ssyndrome(Gustatorysweating)
ComplicationsofSubmandibularGlandExcision
Marginalmandibularnerveinjury;
Lingualnerveinjury;
Hypoglossdnerveinjury
Complicationsofparotidectomy-
Flapnecrosis
Facialnervepalsy
Fluidcollection

Salivaryfistula
Frey'ssyndrome

184.TrueaboutTyphoidulcer:
a)Perforationiscommonin2nd&3rdweekafteronsetoftyphoid
fever
b)Perforationmaypresentatmultiplesite
c)Commonlyinvolvejejunum
d)Commonlyinvolveileum
e)Peritoneallavageiscontraindicatedforperforatedulcer
CorrectAnswer-A:B:D
Answer-(A)Perforationiscommonin2nd&3rdweekafter
onsetoftyphoidfever(B)Perforationmaypresentatmultiple
site(D)Commonlyinvolveileum
CharacteristicFeaturesofTyphoidUlcersMostcommonsiteis
mucosaofsmallintestine(Ileum)inregionoflymphoidpatches
(Payer'spatches)
Ulcersareoval
Ulcersarearrangedlongitudinally(Longitudinalulcers)
Lieinthelongaxisoftheintestinealongtheantimesentericborder
Ulcersmaybesingleormultiple
Ulcersmayerodeandcausecomplications(Bleeding/perforation)

185.Punchedoutulceris/areseenin:
a)Arterialulcer
b)Venousulcer
c)Tubercularulcer
d)Basalcellcarcinoma
e)Gummatousulcer
CorrectAnswer-A:E
Answer-(A)Arterialulcer(E)Gummatousulcer
Puchedoutedge:Itismostlyseeningummatousulcerorinadeep
trophiculcer.
Arterialulcer-Thaeulcertendstobepunchedout
Gummatousulcers,whichoccursintertiarysyphilis,havepunched-
outindolentedge.

186.Whichofthefollowingis/aretrue
regardingrenaltransplantationexcept:
a)Curvilinearincisionismadeinlowerquadrant
b)Transplantedkidneyisplacedinretroperitonealposition
c)Recipientskidneyisfirstremovedfromrenalfossatomake
spacefordonorkidney
d)Anastomosisofexternaliliacarterywithrenalartery
e)Donorrenalveinisanastomosedwithexternaliliacvein
CorrectAnswer-C
Answer-(C)Recipientskidneyisfirstremovedfromrenalfossa
tomakespacefordonorkidney
Alowerqusdrantcurvilinear(Gibson)incisionismade,andtheiliac
vesselsareexposedthrougharetroperitonealapproach.
Therenalarterycanbeanastomosedend-to-endtotheinternaliliac.
Anend-to-sideanastomosisoftherenalarterytotheexternaliliac
arteryismorefrequentlydone.
Theureter,whichiskeptreasonablyshorttoavoidtheriskofdistal
ischaemia,isthenaaastomosedtothebladder.

187.Whichofthefollowingplanof
managementis/arecorrectwithregardto
Cleftlip&cleftpalateinchildren:

a)Unilateralliponly-1yearofage
b)Bilaterallip-1yearofage
c)Bilaterallipshouldberepairedat5month
d)Bothsoft&hardpalateat1yearinonesetting
e)Cleftlipandsoftpalateat5-6months&hardpalateat15-18
monthsofage
CorrectAnswer-C:E
Answer-(C)Bilaterallipshouldberepairedat5month(E)Cleft
lipandsoftpalateat5-6months&hardpalateat15-18months
ofage
Cleftingofthelipand/orpalateisfelttooccuraroundtheeighth
weekofembryogenesis,eitherbyfailureoffusionofthemedial
nasalprocessandthemaxillaryprominenceorbyfailureof
mesodermalmigrationandpenetrationbetweentheepithelialbilayer
oftheface.
Repair
Ruleoftens:Forincreasedanestheticsafety,aninfantshould
1. Be10weeksold.
2. Weigh10pounds.
3. Haveahemoglobinlevelofatleast10mg/dL.

188.Whichofthefollowingis/arefeaturesof
Testiculartorsionwhichisnotpresentin
epididymoorchitis:

a)Rednessofskin
b)Elevationoftestisworsenpain
c)Dopplerultrasoundscanshowabsenceofthebloodsupplyto
theaffectedtestis
d)Fever
e)Dysuria
CorrectAnswer-B:C
Answer-(B)Elevationoftestisworsenpain(C)Doppler
ultrasoundscanshowabsenceofthebloodsupplytothe
affectedtestis
Epididymoorchitis
Theinitialsymptomispain&swellingofepididymisalongwithfever,
malaise&chills
Scrotalwallbecomesred,oedematous&glossy
Testiculartorsion
Thescrotumisswollen&tendon.Thescrotummayalsobenormal
orred&oedematous.
Theonsetofrednessoftheskinandamildpyrexia.
Elevationofthetestisreducesthepaininepididymoorchitisand
makesitworseintorsion.
Dopplerultrasoundscanwillconfirmtheabsenceoftheblood
supplytotheaffectedtestis.

189.Whichofthefollowingis/aretrueabout
primaryhyperparathyroidism:
a)AssociatedwithMENI&II
b)Increaseriskofrenalstone
c)About50gramofglandtissuecanbeleftduringsurgeryto
preventhypocalcemia
d)Causebenignhypertension
e)Mcsporadiccauseishyperplasia
CorrectAnswer-A:B:C
Answer-(A)AssociatedwithMENI&II(B)Increaseriskofrenal
stone(C)About50gramofglandtissuecanbeleftduring
surgerytopreventhypocalcemia
PrimaryhyperparathyroidismisduetoexcessPTHsecretionfroma
singleparathyroidadenoma,hyperplasia,andcarcinoma.
PrimaryhyperparathyroidismMEN1,MEN2A,isolatedfamilialHPT,
andfamlialHPTwithjaw-tumorsyndrome.
Inheritedinanautosomaldominant.
Theclassicquartetofstones,bones,abdominalgroansandpsychic
moansrenalcalculiandcalcinosis,pancreatitisandpsychiatric
disorderdevelops.

190.Suturenumber2-0correspondsto
diameterof:
a)0.21mm
b)0.24mm
c)0.27mm
d)0.30mm
e)0.33mm
CorrectAnswer-D
Answer-(D)0.30mm
Theavailablesizesanddiametersofsutureare:
6-0=0.07mm
5-0=0.10mm
4-0=0.15mm
3-0=0.20mm
2-0=0.30mm
0=0.35mm
1=0.40mm
2=0.5mm

191.Psammomabodiesis/areseenin:
a)Medullarycarcinomaofthyroid
b)Ependymoma
c)Papillarycarcinomathyroid
d)Follicularcarcinomaofthyroid
e)Meningioma
CorrectAnswer-C:E
Answer-(C)Papillarycarcinomathyroid(E)Meningioma
Psammomabodiescanbeseenin:?
Papillarycarcinomaofthyroid.
Renalcellcarcinoma(papillarytype)
Serouscystadenomaofovary
Meningioma
Malignantmesothelioma(Peritoneal&pleural)
Somatostatinoma(Pancreas)
Prolactinomam(Pituitary)
Endometrialpapillaryserouscystadenocarcinoma

192.Whichofthefollowingis/arecorrect
regardingeczemaofbreast&Paget's
diseaseofnipple:

a)Generallyeczemaisunilateralwhereaspaget'sdiseaseis
bilateral
b)Eczemamaybeassociatedwithlactation
c)NipplemaybedestroyedinPaget'sdiseasewhileitnormally
intactineczema
d)InversionofnipplemayoccurinPaget'sdisease
e)Associatedwithbreastlumpineczema
CorrectAnswer-B:C:D
Answer-(B)Eczemamaybeassociatedwithlactation
(C)NipplemaybedestroyedinPaget'sdiseasewhileit
normallyintactineczema(D)Inversionofnipplemayoccurin
Paget'sdisease
Paget'sdiseaseofnipple-
Occursalmostexclusivelyinwomen.
Mayoccuratanyagefrom24to84,althoughitismostcommon
aroundthemenopause.
Almostalwaysassociatedwithunderlyingcarcinomaofthebreast
Unilateral,persistenteczematous-typechangeofthenipplewith
erythemaandscaling.
Itchingorburningsensation.
Destructionofthenipple.

193.Correctcombination(s)ofHerniawith
theircontent:
a)Littrehernia:Meckle'sdiverticulum
b)Richter'shernial-smallintestine
c)Pantaloon-hernialsaconeithersideofinferiorgastricartery
d)Hernia-en-glissade-urinarybladder
e)Maydl'shernia-intestine
CorrectAnswer-A:B:C:D:E
Answer-(A)Littrehernia:Meckle'sdiverticulum(B)Richter's
hernial-smallintestine(C)Pantaloon-hernialsaconeitherside
ofinferiorgastricartery(D)Hernia-en-glissade-urinarybladder
(E)Maydl'shernia-intestine
Richter'sherniaisaherniainwhichthesaccontainsonlyaportion
ofthecircumferenceoftheintestine.
Maydl'sHernia-normal-lookingloopsofintestinearepresentinthe
sac
SlidingHernia(Hernia-en-glissade)-oneithersidebyaportionof
thebladder
Saddle-bag,pantaloonhernia-Thistypeofherniaconsistsoftwo
sacsthatstaddletheinferiorepigastricartery
Littrehernia-AherniacontainingaMeckel'sdiverticulum

194.Trueaboutduodenaldiverticula:
a)MostcommonoftheGITdiverticula
b)Commonlyoccuronmedialborder
c)Oftenpresentinperiampullaryregion
d)Itshouldbetreatedaggressivelyduetohighmalignantpotential
e)Itcancausepancreatitisbyproducingobstructionatthe
openingoftheduct
CorrectAnswer-B:C:E
Ans.(B)Commonlyoccuronmedialborder(C)Oftenpresentin
periampullaryregion(E)Itcancausepancreatitisbyproducing
obstructionattheopeningoftheduct
Diverticulaoftheduodenumrepresentingthesecondmostcommon
sitefordiverticulumformationafterthecolon.
Duodenaldiverticulaoccurtwiceasofteninwomen.
Twothirdstothreefourthsofduodenaldiverticulaarefoundinthe
periampullaryregion.
Majorcomplicationsofduodenaldiverticulaincludeobstructionof
thebiliaryorpancreaticducts.
Hemorrhage;perforation;andrarely,blindloopsyndrome
OnlythosediverticulaassociatedwiththeampullaofVaterare
significantlyrelatedtocomplicationsofcholangitisandpancreatitis.
Surgicaltreatmentisrequiredforcomplications&rarelyfor
persistentsymptoms.

195.Whichofthefollowingistrueabout
esophagusexcept:
a)Serosanotpresent
b)Middle&lowerthirdmadeupofstriatedmuscle
c)Loweroesophagealsphincterisazoneofrelativelyhigh
pressure
d)InachaliacardiaLOStoneisincreasedsothatfoodcannot
passbelow
e)Ithassidetoside&antero-posteriorcurvatures
CorrectAnswer-B
Answer-(B)Middle&lowerthirdmadeupofstriatedmuscle
Epitheliumisstratifiedsquamousnon-keratinizedepithelium.
Muscularismucosacontainsonlylongitudinallayerandnocircular
layer.
Mucosaisthetoughestandstrongestlayer.
Serosaisabsent.
Muscularisexternaismadeupofskeletalmusclefibersonlyinthe
upperthird,smoothmusclesonlyinthelowerthirdandbothtypesof
musclefibersinmiddlethird.Atupperendthelongitudinalcoatsplits
intotwobundlesandthetriangularintervalbetweenthemiscalled
Laimer'stringlewhichisfilledwithcircularmusclefiber

196.Whichofthefollowingincreasesriskof
urinarystones:
a)Increasedurinarycitrate
b)1stCalciumlevelinblood
c)HypervitaminosisA
d)Hypoparathyroidism
e)Decreaseinurinarycolloid
CorrectAnswer-B:C:E
Answer-(B)1stCalciumlevelinblood(C)HypervitaminosisA
(E)Decreaseinurinarycolloid
DeficiencyofvitaminA
Alteredurinarysolutesandcolloids
Decreasedurinarycitrate
Renalinfection
Inadequateurinarydrainageandurinarystasis
Prolongedimmobilization
Hyperparathyroidism

197.Whichofthefollowingistrueabout
digitalrectalexamination:
a)Indexfingerisused
b)InBPHrectalmucosaisfreewhereasinprostatecancer
mucosaofrectumisadhered
c)Puborectalisisfeltanteriorly
d)Analsphincterlengthcannotbeassessed
e)None
CorrectAnswer-A:B
Answer-(A)Indexfingerisused(B)InBPHrectalmucosais
freewhereasinprostatecancermucosaofrectumisadhered
Duringdigitalrectalexaminationinamale,thefingersequentially
palpatesontheanteriorwall,theprostategland,theseminal
vesicles,andtherectovesicalpouch.
Posteriorlyisthehollowofthesacrumandthecoccyx.
Ontheanteriorwallofthefemale,theuterinecervix,theuterine
fundus(ifretroverted),andtherectouterinepouchwillbefelt.
Therectalwallispalpatedformassesandnarrowingofthelumen.

198.AllaretrueaboutBPHexcept:
a)Submucosalglandhyperplasiaintransitionzone
b)Penileurethaelongation
c)Prostaticurethramaybecompressed
d)Lessthan20m1/secondpeakflowrateisstrongindicationfor
TURP
e)BPHoccurduetogrowthinperipheralzoneofprostate
CorrectAnswer-B:D:E
Answer-(B)Penileurethaelongation(D)Lessthan
20m1/secondpeakflowrateisstrongindicationforTURP
(E)BPHoccurduetogrowthinperipheralzoneofprostate
BenignProstaticHyperplasia(BPH)
Prostateitisdividedintotheperipheralzone(PZ),whichliesmainly
posteriorlyandfromwhichmostcarcinomasarise,andacentral
zone(CZ).
Thereisalsoaperiurethraltransitionalzone(TZ),fromwhichmost
benignptostatichyperplasia(BPH)arises.
BPHtypicallyafectsthesubmucousgroupofglandsinthe
transitionalzone.
Aflowrate<10mls-1willbesufiicientformosturologiststo
recommendtreatment.

199.Truestatement(s)aboutlaproscopic
cholecystectomy:
a)Cosmeticallybetter
b)Bleedingismorecommoninopencholecystectomythan
laproscopiccholecystectomy
c)Veressneddleisusedinclosedtechnique
d)Lalprocsopewithattachedvideocameraisinsertedthrough
umbilicalport
e)Previousabdominalsurgeryisabsolutecontraindication
CorrectAnswer-A:C:D
Answer-(A)Cosmeticallybetter(C)Veressneddleisusedin
closedtechnique(D)Lalprocsopewithattachedvideocamera
isinsertedthroughumbilicalport
Initially,asmallincisionismadeintheupperedgeoftheumbillcus.
Withtheclosedtechniqueaspecialhollowinsufflationneedle
(Veressneedle)thatisspring-loadedwitharetractablecuttingouter
sheathisinsertedintotheperitonealcavityandusedforinsufflation.
Bleedinghasbeenthemostcommoncauseforconversiontoan
openprocedure.

200.Bilateralparotidswellingis/areseenin
allexcept:
a)Mump
b)Epstein-Barrvirus
c)Sarcoidosis
d)Brucella
e)Sjogrensyndrome
CorrectAnswer-D
Answer-(D)Brucella
Viralinfections
Mumps
Influenza
Epstein-Barrvirus
CoxsackievirusA
Cytomegalovirus
HIVHCV
Sarcoidosis
Sjogren'ssyndrome
Metabolicdisorders
Diabetesmellitus
Chronicpancreatitis
Hepaticcirrhosis

201.Commontumour(s)foundinanterior
mediastinum:
a)Teratoma
b)Cervicalthymoma
c)Lymphoma
d)Schwannoma
e)Thyroidcarcinoma
CorrectAnswer-A:B:C
Answer-A,TeratomaB,CervicalthymomaC,Lymphoma
Thymoma
Teratoma
Parathyroidadenoma
Bronchogeniccyst
Aneurysms
Lymphoma
Lipoma
Spinallesions
Goitre

202.Trueaboutthyroidtumor:
a)Follicular-lymphaticmetastasismorecommonthanpapillary
b)Papillary-bloodmetastasismorecommonthanfollicular
c)Hurthle-lymphaticspreadiscommon
d)Hurthle-lessaggressivethanfollicularcarcinoma
e)Follicular-invasionofcapsule&vascularspacesincapsular
region
CorrectAnswer-D:E
Answer-D,Hurthle-lessaggressivethanfollicular
carcinomaE,Follicular-invasionofcapsule&vascularspaces
incapsularregion
Hurthlecellcarcinomaisasubtypeoffollicularcarcinoma.
Thetumorcontainsanabundanceofoxyphiliccells,oroncocytes.
Itappearsinanolderagegroup.
Higherchanceofspreadtolymphnodecomparedtofollicular
carcinoma.
Treatmentissurgical.
FollicularCarcinoma-
Microscopically,thereisinvasionofthecapsuleandofthevascular
spacesinthecapsularregion.
Bloodbornemetastasesaremorecommon.

203.Burninvolvingonelowerlimbsinadult
correspondencetoarea:
a)4.5%
b)9%
c)13.5%
d)18%
e)27%
CorrectAnswer-D
Answer-(D)18%



204.Trueaboutphysiologicalhernia
a)HerniationofForegut
b)HerniationofForegut+midgut
c)HerniationofMidgut
d)Goesbackaround4weekafterherniation
e)Goesbackaround10weekoffetusage
CorrectAnswer-C:D:E
Answer-C,HerniationofMidgutD,Goesbackaround4week
afterherniationE,Goesbackaround10weekoffetusage
Itisanaturalphenomenonthatoccursinearlypregnancy.
Itusuallyoccursfromaround6-8weeksupuntil13weekin-utero,
afterwhichthebowelretunstotheabdominalcavity.
Atapproximately10-11weekstheabdomenenlargesandthe
intestinesreturntotheabdominalcavity.
At-8weeksgestationalageandisduetoanumberoffactors
includingRapidgrowthofthecranialendofthemidgut(whichwill
formthesmallintestine).

205.Inguinalhernialsurgerymaybe
complicatedby:
a)Testicularatrophy
b)Urinaryretention
c)Impotence
d)Constipation
e)Pain
CorrectAnswer-A:B:E
Answer-A,TesticularatrophyB,UrinaryretentionE,Pain
Complicationduringsurgery
Injurytoiliacvessel-themostseriousbutrare
Injurytourinarybladder
Earlypostoperctiveperiod
Pain-Painiscommonduetoincisionintheskin&somedegreeof
retractionofstructuressuchasinguinalligamentdownwards6
conjointtendonupwards
Bleeding
Urinaryretentioniscommon,moresoinmales
Abdominaldistension
Intermediate-between3&7ila*
Seroma
Woundinfection
Late
Inguinodynia
Testicularatrophy

206.Indicationofcircumcisionincludes:
a)Hypospadias
b)Epispadias
c)Phimosis
d)Balanitis
e)Balanoposthitis
CorrectAnswer-C:D:E
Answer-C,PhimosisD,BalanitisE,Balanoposthitis
Indication-religious&phimosis
Medicalindicationsforcircumcisioninboysinclude-
1. recurrentattacksofbalanoposthitis
2. recurrenturinarytractinfections
3. Inadults,inabilitytoretractforintercourse,abnormallytight
frenulum,balanitis

207.Allaretrueaboutsquamouscell
carcinomaofskinexcept:
a)Itiscalledmarjolinulcerwhendevelopsinscar
b)Radiotherapymaybeusedintreatment
c)Hematogenousspreadiscommon&occurearly
d)Maydevelopinchroniculcer
e)Lymphaticspreadischiefwayofspreading
CorrectAnswer-C
Answer-(C)Hematogenousspreadiscommon&occurearly
SCCisamalignanttumourofkeratinisingcellsoftheepidermisor
itsappendages.
Alsoarisesfromthestratumbasaleoftheepidermis.
SCCisthesecondmostcommonformofskincancer.
Usuallyaffectstheelderly.
SCCisalsoassociatedwithchronicinflammation.
WhenaSCCappearsinascaritisknownasaMarjolin'ulcer.
AssociatedwithUVlightexposure,chronicinflammationandviral
infection.
"SCCistreatedbywideexcisionorradiotherepy.
Lymphaticspreadisthechiefmethodofspreadeventhoughit
occursrelativelylate.

208.TrueaboutBuergerdisease
a)Affectslargerarteryonly
b)Youngermalesaremorecommonlyaffected
c)Phlebitismigransischaracteristic
d)Coldintolerance
e)Veinsmayinvolved
CorrectAnswer-B:C:E
Answer-B,Youngermalesaremorecommonly
affectedC,PhlebitismigransischaracteristicE,Veinsmay
involved
AlsocalledasThromboangiitisObliterans
Itisainflammatoryocclusivevasculardisorderinvolvingsmalland
mediumsizedarteriesandveinsinupperandlowerextremities.
Itinvolvestibialandradialarteriesandsometimessecondarily
extendingtoveinsandnervesofextremities.
Theclinicalfeaturesofthromboangiitisobliteransincludesatriadof
claudicationoftheaffectedextremity,Raynaud'sphenomenon,and
migratorysuperficialveinthrombophlebitis.

209.Trueaboutulcer:
a)Arterialulcer-painless
b)Venousulcer-penetratesdeepfascia
c)Arterialulcer-punchedout
d)Neuropathiculcer-mayinvolvebone
e)Trophiculcer-Puchedoutedge
CorrectAnswer-C:D:E
Answer-C,Arterialulcer-punchedoutD,Neuropathiculcer-may
involveboneE,Trophiculcer-Puchedoutedge
Puchedoutedge:Itismostlyseeningummatousulcerorinadeep
trophiculcer.
Arterialulcer-Thaeulcertendstobepunchedout
Gummatousulcers,whichoccursintertiarysyphilis,havepunched-
outindolentedge.
Neurogeniculcer-itburrowsdeepinside,mayinvolvebone&also
calledasperforatingulcer.
Venousulcer-Depth-superficial,doesnotpenetratedeepfascia.

210.TrueaboutAmoebicliverabscess:
a)Morecommoninrightlobe
b)Patientsusuallypresentwithfever,chills&upperquadrant
abdominalpain
c)Usuallymultiple
d)Abscesscavitycontainsanchovysauce-likefluid
e)Mostcommonlypresentswithjaundice
CorrectAnswer-A:B:D
Answer-A,MorecommoninrightlobeB,Patientsusually
presentwithfever,chills&upperquadrantabdominal
painD,Abscesscavitycontainsanchovysauce-likefluid
Amebicliverabscessisthemostfrequentextraintestinal
manifestationofEntamoebahistolyticainfection.
Therightlobe(posteriorsuperiorquadrant)oftheliverismore
commonlyaffectedthantheleftlobe.
Theabscesscontainsachocolate-coloredfluidthatresembles
anchovypasteandconsistspredominantlyofnecrotichepatocytes.
Anorexia,fevernightsweats,malaise,coughandweightloss.

This post was last modified on 11 August 2021